Как решать неопределенные интегралы для чайников: как решать, правила вычисления, объяснение Свойства интегралов неопределенных умножение

Содержание

как решать, правила вычисления, объяснение

Одна из операций дифференцирования- нахождение производной (дифференциала) и применении к исследованию функций.

Не менее важной является обратная задача. Если известно поведение функции в окрестностях каждой точки ее определения, то как восстановить функцию в целом, т.е. во всей области ее определения. Эта задача составляет предмет изучения так называемого интегрального исчисления.

Интегрированием называется действие обратное дифференцированию. Или восстановление функции f(х) по данной производной f`(х). Латинское слово “integro” означает – восстановление.

Пример №1 .

Пусть (f(х))’ = 3х 2 . Найдем f(х).

Решение:

Опираясь на правило дифференцирования, нетрудно догадаться, что f(х)=х 3 , ибо

(х 3)’ = 3х 2 Однако, легко можно заметить, что f(х) находится неоднозначно. В качестве f(х) можно взять f(х)= х 3 +1 f(х)= х 3 +2 f(х)= х 3 -3 и др.

Т.к. производная каждой из них равно 3х 2 . (Производная постоянной равна 0). Все эти функции отличаются друг от друга постоянным слагаемым. Поэтому общее решение задачи можно записать в виде f(х)= х 3 +С, где С – любое постоянное действительное число.

Любую из найденных функций f(х) называют первообразной для функции F`(х)= 3х 2

Определение.

Функция F(х) называется первообразной для функции f(х) на заданном промежутке J, если для всех х из этого промежутка F`(х)= f(х). Так функция F(х)=х 3 первообразная для f(х)=3х 2 на (- ∞ ; ∞). Так как, для всех х ~R справедливо равенство: F`(х)=(х 3)`=3х 2

Как мы уже заметили, данная функция имеет бесконечное множество первообразных.

Пример №2.

Функция есть первообразная для всех на промежутке (0; +∞), т.к. для всех ч из этого промежутка, выполняется равенство.

Задача интегрирования состоит в том, чтобы для заданной функции найти все ее первообразные. При решении этой задачи важную роль играет следующее утверждение:

Признак постоянства функции. Если F”(х) = 0 на некотором промежутке I, то функция F – постоянная на этом промежутке.

Доказательство.

Зафиксируем некоторое x 0 из промежутка I. Тогда для любого числа х из такого промежутка в силу формулы Лагранжа можно указать такое число c, заключенное между х и x 0 , что

F(x) – F(x 0) = F”(c)(x-x 0).

По условию F’ (с) = 0, так как с ∈1, следовательно,

F(x) – F(x 0) = 0.

Итак, для всех х из промежутка I

т е. функция F сохраняет постоянное значение.

Все первообразные функции f можно записать с помощью одной формулы, которую называютобщим видом первообразных для функции f. Справедлива следующая теорема (основное свойство первообразных ):

Теорема. Любая первообразная для функции f на промежутке I может быть записана в виде

F(x) + C, (1) где F (х) – одна из первообразных для функции f (x) на промежутке I, а С – произвольная постоянная.

Поясним это утверждение, в котором кратко сформулированы два свойства первообразной:

  1. какое бы число ни поставить в выражение (1) вместо С, получим первообразную для f на промежутке I;
  2. какую бы первообразную Ф для f на промежутке I ни взять, можно подобрать такое число С, что для всех х из промежутка I будет выполнено равенство

Доказательство.

  1. По условию функция F – первообразная для f на промежутке I. Следовательно, F”(х)= f (х) для любого х∈1, поэтому (F(x) + C)” = F”(x) + C”=f(x)+0=f(x), т. е. F(x) + C – первообразная для функции f.
  2. Пусть Ф (х) – одна из первообразных для функции f на том же промежутке I, т. е. Ф”(x) = f (х) для всех x∈I.

Тогда (Ф(x) – F (x))” = Ф”(х)-F’ (х) = f(x)-f(x)=0.

Отсюда следует в. силу признака постоянства функции, что разность Ф(х) – F(х) есть функция, принимающая некоторое постоянное значение С на промежутке I.

Таким образом, для всех х из промежутка I справедливо равенство Ф(х) – F(x)=С, что и требовалось доказать. Основному свойству первообразной можно придать геометрический смысл: графики любых двух первообразных для функции f получаются друг из друга параллельным переносом вдоль оси Оу

Вопросы к конспектам

Функция F(x) является первообразной для функции f(x). Найдите F(1), если f(x)=9×2 – 6x + 1 и F(-1) = 2.

Найдите все первообразные для функции

Для функции (x) = cos2 * sin2x, найдите первообразную F(x), если F(0) = 0.

Для функции найдите первообразную, график которой проходит через точку

Первообразная.

Первообразную легко понять на примере.

Возьмем функцию у = х 3 . Как мы знаем из предыдущих разделов, производной от х 3 является 3х 2:

(х 3)” = 3х 2 .

Следовательно, из функции у = х 3 мы получаем новую функцию: у = 3х 2 .
Образно говоря, функция у = х 3 произвела функцию у = 3х 2 и является ее «родителем». В математике нет слова «родитель», а есть родственное ему понятие: первообразная.

То есть: функция у = х 3 является первообразной для функции у = 3х 2 .

Определение первообразной:

В нашем примере (х 3)” = 3х 2 , следовательно

у = х 3 – первообразная для у = 3х 2 .

Интегрирование.

Как вы знаете, процесс нахождения производной по заданной функции называется дифференцированием. А обратная операция называется интегрированием.

Пример-пояснение :

у = 3х 2 + sin x .

Решение :

Мы знаем, что первообразной для 3х 2 является х 3 .

Первообразной для sin x является –cos x .

Складываем два первообразных и получаем первообразную для заданной функции:

у = х 3 + (–cos x ),

у = х 3 – cos x .

Ответ :
для функции у = 3х 2 + sin x

у = х 3 – cos x .

Пример-пояснение :

Найдем первообразную для функции у = 2 sin x .

Решение :

Замечаем, что k = 2. Первообразной для sin x является –cos x .

Следовательно, для функции у = 2 sin x первообразной является функция у = –2 cos x .
Коэффициент 2 в функции у = 2 sin x соответствует коэффициенту первообразной, от которой эта функция образовалась.

Пример-пояснение :

Найдем первообразную для функции y = sin 2x .

Решение :

Замечаем, что k = 2. Первообразной для sin x является –cos x .

Применяем нашу формулу при нахождении первообразной для функции

y = cos 2x :

1
y = – · (–cos 2x ),
2

cos 2x
y = – —-
2

cos 2x
Ответ : для функции y = sin 2x первообразной является функция y = – —-
2


(4)

Пример-пояснение .

Возьмем функцию из предыдущего примера: y = sin 2x .

Для этой функции все первообразные имеют вид:

cos 2x
y = – —- + C .
2

Пояснение .

Возьмем первую строчку. Читается она так: если функция y = f(x )равна 0, то первообразной для для нее является 1. Почему? Потому что производная единицы равна нулю: 1″ = 0.

В таком же порядке читаются и остальные строчки.

Как выписывать данные из таблицы? Возьмем восьмую строчку:

(-cos x )” = sin x

Пишем вторую часть со знаком производной, затем знак равенства и производную.

Читаем: первообразной для функции sin x является функция -cos x .

Или: функция -cos x является первообразной для функции sin x .

Первообразная функция и неопределённый интеграл

Факт 1. Интегрирование – действие, обратное дифференцированию, а именно, восстановление функции по известной производной этой функции. Восстановленная таким образом функция F (x ) называется первообразной для функции

f (x ).

Определение 1. Функция F (x f (x ) на некотором промежутке X , если для всех значений x из этого промежутка выполняется равенство F “(x )=f (x ), то есть данная функция f (x ) является производной от первообразной функции F (x ). .

Например, функция F (x ) = sin x является первообразной для функции f (x ) = cos x на всей числовой прямой, так как при любом значении икса (sin x )” = (cos x ) .

Определение 2. Неопределённым интегралом функции f (x ) называется совокупность всех её первообразных . При этом употребляется запись

f (x )dx

,

где знак называется знаком интеграла, функция f (x ) – подынтегральной функцией, а f (x )dx – подынтегральным выражением.

Таким образом, если F (x ) – какая-нибудь первообразная для f (x ) , то

f (x )dx = F (x ) +C

где C – произвольная постоянная (константа).

Для понимания смысла множества первообразных функции как неопределённого интеграла уместна следующая аналогия. Пусть есть дверь (традиционная деревянная дверь). Её функция – “быть дверью”. А из чего сделана дверь? Из дерева. Значит, множеством первообразных подынтегральной функции “быть дверью”, то есть её неопределённым интегралом, является функция “быть деревом + С”, где С – константа, которая в данном контексте может обозначать, например, породу дерева. Подобно тому, как дверь сделана из дерева при помощи некоторых инструментов, производная функции “сделана” из первообразной функции при помощи

формулы, которую мы узнали, изучая производную .

Тогда таблица функций распространённых предметов и соответствующих им первообразных (“быть дверью” – “быть деревом”, “быть ложкой” – “быть металлом” и др.) аналогична таблице основных неопределённых интегралов, которая будет приведена чуть ниже. В таблице неопределённых интегралов перечисляются распространённые функции с указанием первообразных, из которых “сделаны” эти функции. В части задач на нахождение неопределённого интеграла даны такие подынтегральные функции, которые без особых услилий могут быть проинтегрированы непосредственно, то есть по таблице неопределённых интегралов. В задачах посложнее подынтегральную функцию нужно предварительно преобразовать так, чтобы можно было использовать табличные интегралы.

Факт 2. Восстанавливая функцию как первообразную, мы должны учитывать произвольную постоянную (константу) C , а чтобы не писать список первообразной с различными константами от 1 до бесконечности, нужно записывать множество первообразных с произвольной константой C , например, так: 5x ³+С . Итак, произвольная постоянная (константа) входит в выражение первообразной, поскольку первообразная может быть функцией, например, 5x ³+4 или 5x ³+3 и при дифференцировании 4 или 3, или любая другая константа обращаются в нуль.

Поставим задачу интегрирования: для данной функции f (x ) найти такую функцию F (x ), производная которой равна f (x ).

Пример 1. Найти множество первообразных функции

Решение. Для данной функции первообразной является функция

Функция F (x ) называется первообразной для функции f (x ), если производная F (x ) равна f (x ), или, что одно и то же, дифференциал F (x ) равен f (x ) dx , т.е.

(2)

Следовательно, функция – первообразная для функции . Однако она не является единственной первообразной для . Ими служат также функции

где С – произвольная постоянная. В этом можно убедиться дифференцированием.

Таким образом, если для функции существует одна первообразная, то для неё существует бесконечное множество первообразных, отличающихся на постоянное слагаемое. Все первообразные для функции записываются в приведённом выше виде. Это вытекает из следующей теоремы.

Теорема (формальное изложение факта 2). Если F (x ) – первообразная для функции f (x ) на некотором промежутке Х , то любая другая первообразная для f (x ) на том же промежутке может быть представлена в виде F (x ) + C , где С – произвольная постоянная.

В следующем примере уже обращаемся к таблице интегралов, которая будет дана в параграфе 3, после свойств неопределённого интеграла. Делаем это до ознакомления со всей таблицей, чтобы была понятна суть вышеизложенного. А после таблицы и свойств будем пользоваться ими при интегрировании во всей полносте.

Пример 2. Найти множества первообразных функций:

Решение. Находим множества первообразных функций, из которых “сделаны” данные функции. При упоминании формул из таблицы интегралов пока просто примите, что там есть такие формулы, а полностью саму таблицу неопределённых интегралов мы изучим чуть дальше.

1) Применяя формулу (7) из таблицы интегралов при n = 3, получим

2) Используя формулу (10) из таблицы интегралов при n = 1/3, имеем

3) Так как

то по формуле (7) при n = -1/4 найдём

Под знаком интеграла пишут не саму функцию f , а её произведение на дифференциал dx . Это делается прежде всего для того, чтобы указать, по какой переменной ищется первообразная. Например,

, ;

здесь в обоих случаях подынтегральная функция равна , но её неопределённые интегралы в рассмотренных случаях оказываются различными. В первом случае эта функция рассматривается как функция от переменной x , а во втором – как функция от z .

Процесс нахождения неопределённого интеграла функции называется интегрированием этой функции.

Геометрический смысл неопределённого интеграла

Пусть требуется найти кривую y=F(x) и мы уже знаем,что тангенс угла наклона касательной в каждой её точке есть заданная функция f(x) абсциссы этой точки.

Согласно геометрическому смыслу производной, тангенс угла наклона касательной в данной точке кривой y=F(x) равен значению производной F”(x) . Значит, нужно найти такую функцию F(x) , для которой F”(x)=f(x) . Требуемая в задаче функция F(x) является первообразной от f(x) . Условию задачи удовлетворяет не одна кривая, а семейство кривых. y=F(x) – одна из таких кривых, а всякая другая кривая может быть получена из неё параллельным переносом вдоль оси Oy .

Назовём график первообразной функции от f(x) интегральной кривой. Если F”(x)=f(x) , то график функции y=F(x) есть интегральная кривая.

Факт 3. Неопределённый интеграл геометрически представлен семеством всех интегральных кривых , как на рисунке ниже. Удалённость каждой кривой от начала координат определяется произвольной постоянной (константой) интегрирования C .

Свойства неопределённого интеграла

Факт 4. Теорема 1. Производная неопределённого интеграла равна подынтегральной функции, а его дифференциал – подынтегральному выражению.

Факт 5. Теорема 2. Неопределённый интеграл от дифференциала функции f (x ) равен функции f (x ) с точностью до постоянного слагаемого , т.е.

(3)

Теоремы 1 и 2 показывают, что дифференцирование и интегрирование являются взаимно-обратными операциями.

Факт 6. Теорема 3. Постоянный множитель в подынтегральном выражении можно выносить за знак неопределённого интеграла , т.е.

Решение интегралов – задача легкая, но только для избранных. Эта статья для тех, кто хочет научиться понимать интегралы, но не знает о них ничего или почти ничего. Интеграл… Зачем он нужен? Как его вычислять? Что такое определенный и неопределенный интегралы? Если единственное известное вам применение интеграла – доставать крючком в форме значка интеграла что-то полезное из труднодоступных мест, тогда добро пожаловать! Узнайте, как решать интегралы и почему без этого никак нельзя обойтись.

Изучаем понятие “интеграл”

Интегрирование было известно еще в Древнем Египте. Конечно, не в современном виде, но все же. С тех пор математики написали очень много книг по этой теме. Особенно отличились Ньютон и Лейбниц , но суть вещей не изменилась. Как понять интегралы с нуля? Никак! Для понимания этой темы все равно понадобятся базовые знания основ математического анализа. Именно эти фундаментальные сведения о Вы найдете у нас в блоге.

Неопределенный интеграл

Пусть у нас есть какая-то функция f(x) .

Неопределенным интегралом функции f(x) называется такая функция F(x) , производная которой равна функции f(x) .

Другими словами интеграл – это производная наоборот или первообразная. Кстати, о том, как читайте в нашей статье.

Первообразная существует для всех непрерывных функций. Также к первообразной часто прибавляют знак константы, так как производные функций, различающихся на константу, совпадают. Процесс нахождения интеграла называется интегрированием.

Простой пример:

Чтобы постоянно не высчитывать первообразные элементарных функций, их удобно свести в таблицу и пользоваться уже готовыми значениями:

Определенный интеграл

Имея дело с понятием интеграла, мы имеем дело с бесконечно малыми величинами. Интеграл поможет вычислить площадь фигуры, массу неоднородного тела, пройденный при неравномерном движении путь и многое другое. Следует помнить, что интеграл – это сумма бесконечно большого количества бесконечно малых слагаемых.

В качестве примера представим себе график какой-нибудь функции. Как найти площадь фигуры, ограниченной графиком функции?

С помощью интеграла! Разобьем криволинейную трапецию, ограниченную осями координат и графиком функции, на бесконечно малые отрезки. Таким образом фигура окажется разделена на тонкие столбики. Сумма площадей столбиков и будет составлять площадь трапеции. Но помните, что такое вычисление даст примерный результат. Однако чем меньше и уже будут отрезки, тем точнее будет вычисление. Если мы уменьшим их до такой степени, что длина будет стремиться к нулю, то сумма площадей отрезков будет стремиться к площади фигуры. Это и есть определенный интеграл, который записывается так:


Точки а и b называются пределами интегрирования.

Бари Алибасов и группа “Интеграл”

Кстати! Для наших читателей сейчас действует скидка 10% на

Правила вычисления интегралов для чайников

Свойства неопределенного интеграла

Как решать неопределенный интеграл? Здесь мы рассмотрим свойства неопределенного интеграла, которые пригодятся при решении примеров.

  • Производная от интеграла равна подынтегральной функции:

  • Константу можно выносить из-под знака интеграла:

  • Интеграл от суммы равен сумме интегралов. Верно также для разности:

Свойства определенного интеграла

  • Знак интеграла изменяется, если поменять местами пределы интегрирования:

  • При любых точках a , b и с :

Мы уже выяснили, что определенный интеграл – это предел суммы. Но как получить конкретное значение при решении примера? Для этого существует формула Ньютона-Лейбница:

Примеры решения интегралов

Ниже рассмотрим несколько примеров нахождения неопределенных интегралов. Предлагаем Вам самостоятельно разобраться в тонкостях решения, а если что-то непонятно, задавайте вопросы в комментариях.

Для закрепления материала посмотрите видео о том, как решаются интегралы на практике. Не отчаиваетесь, если интеграл не дается сразу. Спросите , и они расскажут вам о вычислении интегралов все, что знают сами. С нашей помощью любой тройной или криволинейный интеграл по замкнутой поверхности станет вам по силам.

Определение первообразной.

Первообразной функции f(x) на промежутке (a; b) называется такая функция F(x) , что выполняется равенство для любого х из заданного промежутка.

Если принять во внимание тот факт, что производная от константы С равна нулю, то справедливо равенство . Таким образом, функция f(x) имеет множество первообразных F(x)+C , для произвольной константы С , причем эти первообразные отличаются друг от друга на произвольную постоянную величину.

Определение неопределенного интеграла.

Все множество первообразных функции f(x) называется неопределенным интегралом этой функции и обозначается .

Выражение называют подынтегральным выражением , а f(x) – подынтегральной функцией . Подынтегральное выражение представляет собой дифференциал функции f(x) .

Действие нахождения неизвестной функции по заданному ее дифференциалу называется неопределенным интегрированием, потому что результатом интегрирования является не одна функция F(x) , а множество ее первообразных F(x)+C .

На основании свойств производной можно сформулировать и доказать свойства неопределенного интеграла (свойства первообразной).

Промежуточные равенства первого и второго свойств неопределенного интеграла приведены для пояснения.

Для доказательства третьего и четвертого свойств достаточно найти производные от правых частей равенств:

Эти производные равны подынтегральным функциям, что и является доказательством в силу первого свойства. Оно же используется в последних переходах.

Таким образом, задача интегрирования является обратной задаче дифференцирования, причем между этими задачами очень тесная связь:

  • первое свойство позволяет проводить проверку интегрирования. Чтобы проверить правильность выполненного интегрирования достаточно вычислить производную полученного результата. Если полученная в результате дифференцирования функция окажется равной подынтегральной функции, то это будет означать, что интегрирование проведено верно;
  • второе свойство неопределенного интеграла позволяет по известному дифференциалу функции найти ее первообразную. На этом свойстве основано непосредственное вычисление неопределенных интегралов.

Рассмотрим пример.

Пример.

Найти первообразную функции , значение которой равно единице при х = 1 .

Решение.

Мы знаем из дифференциального исчисления, что (достаточно заглянуть в таблицу производных основных элементарных функций). Таким образом, . По второму свойству . То есть, имеем множество первообразных . При х = 1 получим значение . По условию, это значение должно быть равно единице, следовательно, С = 1 . Искомая первообразная примет вид .

Пример.

Найти неопределенный интеграл и результат проверить дифференцированием.

Решение.

По формуле синуса двойного угла из тригонометрии , поэтому

Задания на определенный интеграл. Интегралы – что это, как решать, примеры решений и объяснение для чайников. Правила вычисления интегралов для чайников

Данный калькулятор позволяет решить определенный интеграл онлайн. По сути, вычисление определенного интеграла – это нахождение числа, которое равно площади под графиком функции. Для решения необходимо задать границы интегрирования и интегрируемую функцию. После интегрирования система найдет первообразную для заданной функции, вычислит её значения в точках границах интегрирования, найдет их разность, что и будет являться решением определенного интеграла. Чтобы решить неопределенный интеграл вам необходимо воспользоваться похожим онлайн калькулятором, который находится на нашем сайте по ссылке – Решить неопределенный интеграл .

Мы позволяем вычислить определенный интеграл онлайн быстро и надежно. Вы получите всегда верное решение. Причем для табличных интегралов ответ будет представляться в классическом виде, то есть выражаться через известные константы, такие как число “пи”, “экспонента” и т.д. Все вычисления полностью бесплатны и не требуют регистрации. Решая определенный интеграл у нас, вы избавите себя от трудоемких и сложных вычислений, либо решив интеграл самостоятельно – вы сможете проверить полученное вами решение.

Онлайн сервис на сайт позволяет находить решение определенного интеграла онлайн . Решение проводится автоматически на сервере и в течении нескольких секунд пользователю выдается результат. Все онлайн сервисы на сайте абсолютно бесплатны, а решение выдается в удобном и понятном виде. Также нашим преимуществом является, что мы предоставляем возможность пользователю ввести границы интегрирования, в том числе и пределы интегрирования: минус и плюс бесконечность. Таким образом, решить определенный интеграл становится просто, быстро и качественно. Важно, что сервер позволяет вычислять определенные интегралы онлайн сложных функций, решение которых на иных онлайн-сервисах часто является невозможным ввиду несовершенства их систем. Мы предоставляем очень простой и интуитивно понятный механизм для ввода функций и возможность выбора переменной интегрирования, для чего вам не приходится переводить заданную в одной переменной функцию в другую, исключая связанные с этим ошибки и опечатки. Также на странице даны ссылки на теоретические статьи и таблицы по решению определенных интегралов. Всё в совокупоности позволит вам вычислять определенный интеграл онлайн очень быстро и при желании найти и разобраться с теорией решения определенных интегралов. На http://сайт вы также можете переходить на другие сервисы: онлайн решение пределов, производных, суммы рядов. Перейти же на вкладку решения неопределенных интегралов онлайн совсем просто – ссылка находится в ряду среди полезных ссылок. Более того, сервис постоянно совершенствуется и развивается, и с каждым днем появляются всё новые и новые возможности и усовершенствования. Решайте определенные интегралы вместе с нами! Все онлайн сервисы доступны даже незарегистрировшимся пользователям и абсолютно бесплатны.

Решая определенный интеграл у нас вы можете проверить своё собственное решение или избавиться от излишних трудоемких вычислений и довериться высокотехнологичной автоматизированной машине. Вычисляемая на сервисе точность удовлетворит практически любые инженерные нормы. Часто для многих табличных определенных интегралов результат выдается в точном выражении (используя общеизвестные константы и неэлементарные функции).

В каждой главе будут и задачи для самостоятельного решения, к которым можно посмотреть ответы.

Понятие определённого интеграла и формула Ньютона-Лейбница

Определённым интегралом от непрерывной функции f (x ) на конечном отрезке [a , b ] (где ) называется приращение какой-нибудь её первообразной на этом отрезке. (Вообще, понимание заметно облегчится, если повторить тему неопределённого интеграла) При этом употребляется запись

Как видно на графиках внизу (приращение первообразной функции обозначено ), определённый интеграл может быть как положительным, так и отрицательным числом (Вычисляется как разность между значением первообразной в верхнем пределе и её же значением в нижнем пределе, т. е. как F (b ) – F (a )).

Числа a и b называются соответственно нижним и верхним пределами интегрирования, а отрезок [a , b ] – отрезком интегрирования.

Таким образом, если F (x ) – какая-нибудь первообразная функция для f (x ), то, согласно определению,

(38)

Равенство (38) называется формулой Ньютона-Лейбница . Разность F (b ) – F (a ) кратко записывают так:

Поэтому формулу Ньютона-Лейбница будем записывать и так:

(39)

Докажем, что определённый интеграл не зависит от того, какая первообразная подынтегральной функции взята при его вычислении. Пусть F (x ) и Ф(х ) – произвольные первообразные подынтегральной функции. Так как это первообразные одной и той же функции, то они отличаются на постоянное слагаемое: Ф(х ) = F (x ) + C . Поэтому

Тем самым установлено, что на отрезке [a , b ] приращения всех первообразных функции f (x ) совпадают.

Таким образом, для вычисления определённого интеграла необходимо найти любую первообразную подынтегральной функции, т.е. сначала следует найти неопределённый интеграл. Постоянная С из последующих вычислений исключается. Затем применяется формула Ньютона-Лейбница: в первообразную функцию подставляется значение верхнего предела b , далее – значение нижнего предела a и вычисляется разность F(b) – F(a) . Полученное число и будет определённым интегралом. .

При a = b по определению принимается

Пример 1.

Решение. Сначала найдём неопределённый интеграл:

Применяя формулу Ньютона-Лейбница к первообразной

(при С = 0), получим

Однако при вычислении определённого интеграла лучше не находить отдельно первообразную, а сразу записывать интеграл в виде (39).

Пример 2. Вычислить определённый интеграл

Решение. Используя формулу

Найти определённый интеграл самостоятельно, а затем посмотреть решение

Свойства определённого интеграла

Теорема 2. Величина определённого интеграла не зависит от обозначения переменной интегрирования , т.е.

(40)

Пусть F (x ) – первообразная для f (x ). Для f (t ) первообразной служит та же функция F (t ), в которой лишь иначе обозначена независимая переменная. Следовательно,

На основании формулы (39) последнее равенство означает равенство интегралов

Теорема 3. Постоянный множитель можно выносить за знак определённого интеграла , т.е.

(41)

Теорема 4. Определённый интеграл от алгебраической суммы конечного числа функций равен алгебраической сумме определённых интегралов от этих функций , т.е.

(42)

Теорема 5. Если отрезок интегрирования разбит на части, то определённый интеграл по всему отрезку равен сумме определённых интегралов по его частям , т.е. если

(43)

Теорема 6. При перестановке пределов интегрирования абсолютная величина определённого интеграла не меняется, а изменяется лишь его знак , т.е.

(44)

Теорема 7 (теорема о среднем). Определённый интеграл равен произведению длины отрезка интегрирования на значение подынтегральной функции в некоторой точке внутри его , т.е.

(45)

Теорема 8. Если верхний предел интегрирования больше нижнего и подынтегральная функция неотрицательна (положительна), то и определённый интеграл неотрицателен (положителен), т.е. если


Теорема 9. Если верхний предел интегрирования больше нижнего и функции и непрерывны, то неравенство

можно почленно интегрировать , т.е.

(46)

Свойства определённого интеграла позволяют упрощать непосредственное вычисление интегралов.

Пример 5. Вычислить определённый интеграл

Используя теоремы 4 и 3, а при нахождении первообразных – табличные интегралы (7) и (6), получим


Определённый интеграл с переменным верхним пределом

Пусть f (x ) – непрерывная на отрезке [a , b ] функция, а F (x ) – её первообразная. Рассмотрим определённый интеграл

(47)

а через t обозначена переменная интегрирования, чтобы не путать её с верхней границей. При изменении х меняется и опредёленный интеграл (47), т.е. он является функцией верхнего предела интегрирования х , которую обозначим через Ф (х ), т.е.

(48)

Докажем, что функция Ф (х ) является первообразной для f (x ) = f (t ). Действительно, дифференцируя Ф (х ), получим

так как F (x ) – первообразная для f (x ), а F (a ) – постояная величина.

Функция Ф (х ) – одна из бесконечного множества первообразных для f (x ), а именно та, которая при x = a обращается в нуль. Это утверждение получается, если в равенстве (48) положить x = a и воспользоваться теоремой 1 предыдущего параграфа.

Вычисление определённых интегралов методом интегрирования по частям и методом замены переменной

где, по определению, F (x ) – первообразная для f (x ). Если в подынтегральном выражении произвести замену переменной

то в соответствии с формулой (16) можно записать

В этом выражении

первообразная функция для

В самом деле, её производная, согласно правилу дифференцирования сложной функции , равна

Пусть α и β – значения переменной t , при которых функция

принимает соответственно значения a и b , т.е.

Но, согласно формуле Ньютона-Лейбница, разность F (b ) – F (a ) есть

Примеры вычисления неопределённых интегралов

Вычисление интеграла по таблице

Интегрирование подстановкой:

Примеры вычисления интегралов

Основная формула Ньютона – Лейбница

Вычисления подстановкой

Глава 4 Дифференциальные уравнения.

Дифференциальным уравнением называется уравнение, связывающее между собой независимую переменную х , искомую функции у и ее производные или дифференциалы.

Символически дифференцированное уравнение записывается так:

Дифференциальное уравнение называется обыкновенным , если искомая функция зависит от одного независимого переменного.

Порядком дифференциального уравнения называется порядок старшей производной (или дифференциала), входящей в данное уравнение.

Решением (или интегралом ) дифференциального уравнения называется такая функция, которая обращает это уравнение в тождество.

Общим решением (или общим интегралом ) дифференциального уравнения называется такое решение, в которое входит столько независимых произвольных постоянных, каков порядок уравнения. Так, общее решение дифференциального уравнения первого порядка содержит одну произвольную постоянную.

Частным решением дифференциального уравнения называется решение, полученное из общего при различных числовых значениях произвольных постоянных. Значения произвольных постоянных находятся при определенных начальных значениях аргумента и функции.

График частного решения дифференциального уравнения называется интегральной кривой.

Общему решению дифференциального уравнения соответствует совокупность (семейство) всех интегральных кривых.

Дифференциальным уравнением первого порядка называется уравнение, в которое входят производные (или дифференциалы) не выше первого порядка.

Дифференциальным уравнением с разделяющимися переменными называется уравнение вида

Для решения этого уравнения нужно сначала разделить переменные:

а затем проинтегрировать обе части полученного равенства:

1. Найти общее решение уравнения

o Разделив переменные имеем

Интегрируя обе части полученного уравнения:

Так как произвольная постоянная С может принимать любые числовые значения, то для удобства дальнейших преобразований вместо C мы написали (1/2) lnC. Потенцируя последнее равенство получим

Это и есть общее решение данного уравнения.

Литература

В. Г. Болтянский, Что такое дифференцирование, «Популярные лекции по математике»,

Выпуск 17, Гостехиздат 1955 г., 64 стр.

В. А. Гусев, А. Г. Мордкович «Математика»

Г. М. Фихтенгольц «Курс дифференциального и интегрального исчисления», том 1

В. М. Бородихин, Высшая математика, учеб. пособие, ISBN 5-7782-0422-1.

Никольский С. М. Глава 9. Определенный интеграл Римана // Курс математического анализа. – 1990. – Т. 1.

Ильин В. А., Позняк, Э. Г. Глава 6. Неопределенный интеграл // Основы математического анализа. – 1998. – Т. 1. – (Курс высшей математики и математической физики).

Демидович Б.П. Отдел 3. Неопределенный интеграл // Сборник задач и упражнений по математическому анализу. – 1990. – (Курс высшей математики и математической физики).

Валуцэ И.И., Дилигул Г.Д. Математика для техникумов на базе средней школы: Учеб.пособие-2-е изд.перераб. и доп. М.6Наука. 1989

Колягин Ю.М. Яковлев Г.Н. математика для техникумов. Алгебра и начала анализа 1 и 2 часть. Издательство «Наукка» М., 1981г.

Щипачев В.С. Задачи по высшей математике: Учеб. Пособие для вузов. Высш. Шк. 1997г.

Богомолов Н.В практические занятия по математике: учеб. Пособие для техникумов. Высш. Шк 1997г.

Решение интегралов – задача легкая, но только для избранных. Эта статья для тех, кто хочет научиться понимать интегралы, но не знает о них ничего или почти ничего. Интеграл… Зачем он нужен? Как его вычислять? Что такое определенный и неопределенный интегралы?

Если единственное известное вам применение интеграла – доставать крючком в форме значка интеграла что-то полезное из труднодоступных мест, тогда добро пожаловать! Узнайте, как решать простейшие и другие интегралы и почему без этого никак нельзя обойтись в математике.

Изучаем понятие « интеграл»

Интегрирование было известно еще в Древнем Египте. Конечно, не в современном виде, но все же. С тех пор математики написали очень много книг по этой теме. Особенно отличились Ньютон и Лейбниц , но суть вещей не изменилась.

Как понять интегралы с нуля? Никак! Для понимания этой темы все равно понадобятся базовые знания основ математического анализа. Сведения о , необходимые и для понимания интегралов, уже есть у нас в блоге.

Неопределенный интеграл

Пусть у нас есть какая-то функция f(x) .

Неопределенным интегралом функции f(x) называется такая функция F(x) , производная которой равна функции f(x) .

Другими словами интеграл – это производная наоборот или первообразная. Кстати, о том, как читайте в нашей статье.


Первообразная существует для всех непрерывных функций. Также к первообразной часто прибавляют знак константы, так как производные функций, различающихся на константу, совпадают. Процесс нахождения интеграла называется интегрированием.

Простой пример:

Чтобы постоянно не высчитывать первообразные элементарных функций, их удобно свести в таблицу и пользоваться уже готовыми значениями.

Полная таблица интегралов для студентов


Определенный интеграл

Имея дело с понятием интеграла, мы имеем дело с бесконечно малыми величинами. Интеграл поможет вычислить площадь фигуры, массу неоднородного тела, пройденный при неравномерном движении путь и многое другое. Следует помнить, что интеграл – это сумма бесконечно большого количества бесконечно малых слагаемых.

В качестве примера представим себе график какой-нибудь функции.


Как найти площадь фигуры, ограниченной графиком функции? С помощью интеграла! Разобьем криволинейную трапецию, ограниченную осями координат и графиком функции, на бесконечно малые отрезки. Таким образом фигура окажется разделена на тонкие столбики. Сумма площадей столбиков и будет составлять площадь трапеции. Но помните, что такое вычисление даст примерный результат. Однако чем меньше и уже будут отрезки, тем точнее будет вычисление. Если мы уменьшим их до такой степени, что длина будет стремиться к нулю, то сумма площадей отрезков будет стремиться к площади фигуры. Это и есть определенный интеграл, который записывается так:


Точки а и b называются пределами интегрирования.


« Интеграл»

Кстати! Для наших читателей сейчас действует скидка 10% на

Правила вычисления интегралов для чайников

Свойства неопределенного интеграла

Как решить неопределенный интеграл? Здесь мы рассмотрим свойства неопределенного интеграла, которые пригодятся при решении примеров.

  • Производная от интеграла равна подынтегральной функции:

  • Константу можно выносить из-под знака интеграла:

  • Интеграл от суммы равен сумме интегралов. Верно также для разности:

Свойства определенного интеграла

  • Знак интеграла изменяется, если поменять местами пределы интегрирования:

  • При любых точках a , b и с :

Мы уже выяснили, что определенный интеграл – это предел суммы. Но как получить конкретное значение при решении примера? Для этого существует формула Ньютона-Лейбница:

Примеры решения интегралов

Ниже рассмотрим неопределенный интеграл и примеры с решением. Предлагаем самостоятельно разобраться в тонкостях решения, а если что-то непонятно, задавайте вопросы в комментариях.


Для закрепления материала посмотрите видео о том, как решаются интегралы на практике. Не отчаиваетесь, если интеграл не дается сразу. Обратитесь в профессиональный сервис для студентов, и любой тройной или криволинейный интеграл по замкнутой поверхности станет вам по силам.

как вычислить интеграл с заменой переменной, его свойства и методы вычисления с подробным решением

Решение интегралов – задача легкая, но только для избранных. Эта статья для тех, кто хочет научиться понимать интегралы, но не знает о них ничего или почти ничего. Интеграл… Зачем он нужен? Как его вычислять? Что такое определенный и неопределенный интегралы?

Если единственное известное вам применение интеграла – доставать крючком в форме значка интеграла что-то полезное из труднодоступных мест, тогда добро пожаловать! Узнайте, как решать простейшие и другие интегралы и почему без этого никак нельзя обойтись в математике.

Интегралы для чайников: как решать, правила вычисления, объяснение

Ежедневная рассылка с полезной информацией для студентов всех направлений – на нашем телеграм-канале.

Изучаем понятие «интеграл»

Интегрирование было известно еще в Древнем Египте. Конечно, не в современном виде, но все же. С тех пор математики написали очень много книг по этой теме. Особенно отличились Ньютон и Лейбниц, но суть вещей не изменилась.

Как понять интегралы с нуля? Никак! Для понимания этой темы все равно понадобятся базовые знания основ математического анализа. Сведения о пределах и производных, необходимые и для понимания интегралов, уже есть у нас в блоге.

Неопределенный интеграл

Пусть у нас есть какая-то функция f(x). Неопределенным интегралом функции f(x) называется такая функция F(x), производная которой равна функции f(x).

Другими словами интеграл – это производная наоборот или первообразная.

Первообразная существует для всех непрерывных функций. Также к первообразной часто прибавляют знак константы, так как производные функций, различающихся на константу, совпадают. Процесс нахождения интеграла называется интегрированием.

Простой пример:

Чтобы постоянно не высчитывать первообразные элементарных функций, их удобно свести в таблицу и пользоваться уже готовыми значениями.

Полная таблица интегралов для студентов

Определенный интеграл

Имея дело с понятием интеграла, мы имеем дело с бесконечно малыми величинами. Интеграл поможет вычислить площадь фигуры, массу неоднородного тела, пройденный при неравномерном движении путь и многое другое. Следует помнить, что интеграл – это сумма бесконечно большого количества бесконечно малых слагаемых.

В качестве примера представим себе график какой-нибудь функции.

Как найти площадь фигуры, ограниченной графиком функции? С помощью интеграла! Разобьем криволинейную трапецию, ограниченную осями координат и графиком функции, на бесконечно малые отрезки. Таким образом фигура окажется разделена на тонкие столбики. Сумма площадей столбиков и будет составлять площадь трапеции.

Но помните, что такое вычисление даст примерный результат. Однако чем меньше и уже будут отрезки, тем точнее будет вычисление. Если мы уменьшим их до такой степени, что длина будет стремиться к нулю, то сумма площадей отрезков будет стремиться к площади фигуры.

Это и есть определенный интеграл, который записывается так:

Правила вычисления интегралов для чайников

Свойства неопределенного интеграла

Как решить неопределенный интеграл? Здесь мы рассмотрим свойства неопределенного интеграла, которые пригодятся при решении примеров.

Производная от интеграла равна подынтегральной функции:

Константу можно выносить из-под знака интеграла:

Интеграл от суммы равен сумме интегралов. Верно также для разности:

Свойства определенного интеграла

Знак интеграла изменяется, если поменять местами пределы интегрирования:

  • При любых точках a, b и с:

Как считать определенный интеграл? С помощью формулы Ньютона-Лейбница.

Мы уже выяснили, что определенный интеграл – это предел суммы. Но как получить конкретное значение при решении примера? Для этого существует формула Ньютона-Лейбница:

Примеры решения интегралов

Ниже рассмотрим неопределенный интеграл и примеры с решением. Предлагаем самостоятельно разобраться в тонкостях решения, а если что-то непонятно, задавайте вопросы в х.

Для закрепления материала посмотрите видео о том, как решаются интегралы на практике. Не отчаиваетесь, если интеграл не дается сразу. Обратитесь в профессиональный сервис для студентов, и любой тройной или криволинейный интеграл по замкнутой поверхности станет вам по силам.

Источник: https://Zaochnik-com.ru/blog/integraly-dlya-chajnikov-kak-reshat-pravila-vychisleniya-obyasnenie/

Первообразная и неопределенный интеграл, их свойства

Для начала, дадим определение понятиям, которые будут использоваться в данном разделе. В первую очередь это первообразная функции. Для этого введем константу C.

Первообразная функции f(x) на промежутке (a; b) это такая функция F(x), при которое формула F'(x)=f(x) превращается в равенство для любого x из заданного промежутка.

Следует учитывать тот факт, что производная от константы C будет равна нулю, что позволяет нам считать верным следующее равенство F(x)+C’=f(x).

Получается, что функция f(x) имеет множество первообразных F(x)+C, для произвольной константы C. Эти первообразные отличаются друг от друга на произвольную постоянную величину.

Определение неопределенного интеграла

Все множество первообразных функции f(x) можно назвать неопределенным интегралом этой функции. С учетом этого формула будет иметь вид ∫f(x)dx=F(x)+C. При этом, выражение f(x)dx является подынтегральным выражением, а f(x) – это подынтегральная функция. Подынтегральное выражение представляет собой дифференциал функции f(x).

Имея заданный дифференциал функции, мы можем найти неизвестную функцию.

Результатом неопределенного интегрирования будет не одна функция F(x), а множество ее первообразных F(x)+C.

Зная свойства производной, мы можем сформулировать и доказать свойства неопределенного интеграла (свойства первообразной).

∫f(x)dx’=F(x)+C’=f(x)

Производная результата интегрирования равна подынтегральной функции.

∫d(F(x))=∫F'(x)dx=∫f(x)dx=F(x)+C

Неопределенный интеграл дифференциала функции равен сумме самой функции и произвольной константы.

∫k·f(x)dx=k·∫f(x)dx, где k – произвольная константа. Коэффициент можно выносить за знак неопределенного интеграла.

Неопределенный интеграл суммы/разности функций равен сумме/разности неопределенных интегралов функций.

∫f(x)±g(x))dx=∫f(x)dx±∫g(x)dx

Промежуточные равенства первого и второго свойств неопределенного интеграла мы привели в качестве пояснения.

Для того, чтобы доказать третье и четвертое свойства, необходимо найти производные от правых частей равенств:

k·∫f(x)dx’=k·∫d(x)dx’=k·f(x)∫f(x)dx±∫g(x)dx’=∫f(x)dx’±∫g(x)dx’=f(x)±g(x)

Производные правых частей равенств равны подынтегральным функциям, что является доказательством первого свойства. Его же мы используем в последних переходах.

Как видите, задача интегрирования представляет собой обратный процесс по отношению к задаче дифференцирования. Обе эти задачи тесно связаны между собой.

Первое свойство может быть использовано для проведения проверки интегрирования. Для проверки нам достаточно вычислить производную полученного результата. Если полученная функция будет равна подынтегральной функции, то интегрирование проведено верно.

Благодаря второму свойству по известному дифференциалу функции мы можем найти ее первообразную и использовать ее для вычисления неопределенного интеграла.

Рассмотрим пример.

Пример 1

Найдем первообразную функции f(x)=1x, значение которой равно единице при х=1.

Решение

Используя таблицу производных основных элементарных функций получаем:

  • d(ln x)=(ln x)’dx=dxx=f(x)dx∫f(x)dx=∫dxx=∫d(ln(x))

Используя второе свойство ∫d(ln(x))=ln(x)+C, мы получаем множество первообразных ln(x)+C. При х=1 получим значение ln(1)+C=0+C=C. Согласно условию задачи, это значение должно быть равно единице, следовательно, С = 1. Искомая первообразная примет вид  ln(x)+1.

Ответ: f(x)=1x=ln(x)+1

Пример 2

Необходимо найти неопределенный интеграл ∫2sinx2cosx2dx и проверить результат вычисления дифференцированием.

Решение

Используем для проведения вычислений формулу синуса двойного угла из курса тригонометрии 2sinx2cosx2=sin x, получим ∫2sinx2cosx2dx=∫sin xdx.

Используем таблицу производных для тригонометрических функций, получим:

  • d(cos x)=cos x’dx=-sin xdx⇒sin xdx=-d(cos x)
  • То есть, ∫sin xdx=∫(-d(cos x))
  • Используя третье свойство неопределенного интеграла, мы можем записать ∫-d(cos x)=-∫d(cos x).
  • По второму свойству получаем -∫d(cos x)=-(cos x+C)
  • Следовательно, ∫2sin x2cosx2dx=-cos x-C.
  • Проверим полученный результат дифференцированием.
  • Продифференцируем полученное выражение:
    -cos x-C’=-(cos x)’-(C)’=-(-sin x)=sin x=2sinx2cosx2

В результате проверки мы получили подынтегральную функцию. Это значит, что интегрирование было проведено нами верно. Для осуществления последнего перехода мы использовали формулу синуса двойного угла.

Ответ: ∫2sin x2cosx2dx=-cos x-C

Если таблицу производных основных элементарных функций переписать в виде дифференциалов, то из нее по второму свойству неопределенного интеграла можно составить таблицу первообразных.

Источник: https://Zaochnik.com/spravochnik/matematika/integraly-integrirovanie/pervoobraznaja-i-neopredelennyj-integral-ih-svojst/

Интегралы для чайников с примерами решения

Содержание:

  1. Первообразная и неопределенный интеграл
  2. Пример с решением:
  3. Таблица интегралов
  4. Некоторые свойства неопределенного интеграла
  5. Постановка задачи. Нижняя и верхняя интегральные суммы
  6. Определенный интеграл. Теорема о существовании определенного интеграла

Первообразная и неопределенный интеграл

Я рассматривала такую задачу: дана функция требуется найти ее производную, т. е. функцию

В этой главе мы будем рассматривать обратную задачу: дана функция требуется найти такую функцию производная которой равна

Определение 1. Функция называется первообразной от функции на отрезке если во всех точках этого отрезка выполняется равенство

По этой ссылке вы найдёте полный курс лекций по высшей математике:

Пример с решением:

Найти первообразную от функции

Из определения первообразной следует, что функция является первообразной, так как

Легко видеть, что если для данной функции существует первообразная, то эта первообразная не является единственной. Так, в предыдущем примере можно было взять в качестве первообразных следующие функции: или вообще (где — произвольная постоянная), так как С другой стороны, можно доказать, что функциями вида исчерпываются все первообразные от функции

Это вытекает из следующей теоремы.

Теорема. Если — две первообразные от функции на отрезке то разность между ними равна постоянному числу.

Доказательство. В силу определения первообразной имеем

при любом значении на отрезке

Обозначим

Тогда на основании равенств (1) будет или при любом значении на отрезке Но из равенства следует, что есть постоянная.

Действительно, применим теорему Лагранжа (см. § 2 гл. IV) к функции которая, очевидно, непрерывна и дифференцируема на отрезке Какова бы ни была точка на отрезке мы имеем в силу теоремы Лагранжа где

Так как , или

Таким образом, функция в любой точке отрезка сохраняет значение а это и значит, что функция является постоянной на отрезке Обозначая постоянную через из равенств (2) и (3) получаем

Из доказанной теоремы следует, что если для данной функции найдена какая-нибудь одна первообразная тo любая другая первообразная для имеет вид где

Определение 2. Если функция является первообразной для то выражение называется неопределенным интегралом от функции и обозначается символом

Таким образом, по определению, если При этом функцию называют подынтегральной функцией, — подынтегральным выражением, знак — знаком интеграла.

Таким образом, неопределенный интеграл представляет собой семейство функций

С геометрической точки зрения неопределенный интеграл представляет совокупность (семейство) кривых, каждая из которых получается путем сдвига одной из кривых параллельно самой себе вверх или вниз, т. е. вдоль оси

Естественно возникает вопрос: для всякой ли функции существуют первообразные (а значит, и неопределенный интеграл)? Оказывается, что не доя всякой. Заметим, однако, без доказательства, что если функция непрерывна на отрезке то для этой функции существует первообразная (а значит, и неопределенный интеграл).

Выяснению методов, с помощью которых находятся первообразные и неопределенные интегралы от некоторых классов элементарных функций, посвящён этот раздел статьи.

Нахождение первообразной для данной функции называется интегрированием функции

Заметим следующее: если производная от элементарной функции всегда является элементарной функцией, то первообразная от элементарной функции может оказаться и не представимой с помощью конечного числа элементарных функций. К этому вопросу мы вернемся в конце данной главы.

Возможно вам будут полезны данные страницы:

Из определения 2 следует:

1. Производная от неопределенного интеграла равна подынтегральной функции, т. е. если то и

Последнее равенство нужно понимать в том смысле, что производная от любой первообразной равна подынтегральной функции.

2. Дифференциал от неопределенного интеграла равен подынтегральному выражению:

Это получается на основании формулы (4).

3. Неопределенный интеграл от дифференциала некоторой функции равен этой функции плюс произвольная постоянная:

Справедливость последнего равенства легко проверить дифференцированием (дифференциалы от обеих частей равенства равный

Таблица интегралов

Прежде чем приступить к изложению методов интегрирования, приведем таблицу интегралов от простейших функций.

Непосредственно из определения 2 § 1 и таблицы производных (§ 15 гл. III) вытекает таблица интегралов. (Справедливость написанных в ней равенств легко проверить дифференцированием, т. е. установить, что производная от правой части равняется подынтегральной функции.)

1. (Здесь и и последующих формулах под понимается произвольная постоянная.)

Замечание. В таблице производных (§ 15 гл. Ill) нет формул, соответствующих формулам 7, 8, 1Г, 12, 13′ и 14. Однако справедливость последних также легко устанавливается с помощью дифференцирования.

В случае формулы 7 имеем

следовательно,

В случае формулы 8

следовательно,

В случае формулы 12

следовательно,

Отметим, что последняя формула будет следовать также из общих результатов § 9. В случае формулы 14

следовательно,

Эта формула также будет следовать из общих результатов § 10.

Некоторые свойства неопределенного интеграла

Теорема 1. Неопределенный интеграл от алгебраической суммы двух или нескольких функций равен алгебраической сумме их интегралов:

Для доказательства найдем производные от левой и правой частей этого равенства. На основании равенства (4) § 1 находим

Таким образом, производные от левой и правой частей равенства (1) равны между собой, т. е. производная от любой первообразной, стоящая в левой части, равняется производной от любой функции, стоящей в правой части равенства.

Следовательно, по теореме § 1 любая функция, стоящая в левой части равенства (1), отличается от любой функции, стоящей в правой части равенства (1), на постоянное слагаемое. В этом смысле и нужно понимать равенство (1).

Теорема 2. Постоянный множитель можно выносить за знак интеграла, т. е. если то

Для доказательства равенства (2) найдем производные от левой и правой его частей:

Производные от правой и левой частей равны, следовательно, как и в равенстве (1), разность двух любых функций, стоящих слева и справа, есть постоянная. В этом смысле и следует понимать равенство (2).

При вычислении неопределенных интегралов бывает полезно иметь в виду следующие правила.

1. Если

то

Действительно, дифференцируя левую и правую части равенства (3), получим

Производные от правой и левой частей равны, что и требозалось доказать.

II. Если

то

Если

то

Равенства (4) и (5) доказываются дифференцированием правой и левой частей равенств.

Пример 1.

Пример 2.

Пример 3.

Пример 4.

Пример 5.

Постановка задачи. Нижняя и верхняя интегральные суммы

Мощным средством исследования в математике, физике, механике и других дисциплинах является определенный интеграл—одно из основных понятий математического анализа. Вычисление площадей, ограниченных кривыми, длин дуг, объемов, работы, скорости, пути, моментов инерции и т. д. сводится к вычислению определенного интеграла.

Пусть на отрезке задана непрерывная функция (рис. 210 и 211). Обозначим через и ее наименьшее и наибольшее значения на этом отрезке. Разобьем отрезок на частей точками деления причем

и положим Обозначим, далее, наименьшее и наибольшее значения функции на отрезке через и на отрезке через и ..на отрезке через и Составим суммы

Сумму называют нижней интегральной суммой, а сумму —верхней интегральной суммой.

Если то нижняя инте;ральная сумма численно равняется площади «вписанной ступенчатой фигуры» ограниченной «вписанной» ломаной, верхняя интегральная сумма численно равняется площади «описанной ступенчатой фигуры»

ограниченной «описанной» ломаной.

Отметим некоторые свойства верхних и нижних интегральных сумм.

а) Так как для любого то на основании формул (1) и (2) имеем

(Знак равенства будет только в случае, если

б) Так как где —наименьшее значение на то

Итак,

в) Так как где — наибольшее значение на то

Итак,

Соединяя вместе полученные неравенства, имеем

Если то последнее неравенство имеет простой геометрический смысл (рис. 212), так как произведения и соответственно численно равны площадям «вписанного» прямоугольника и «описанного» прямоугольника

Определенный интеграл. Теорема о существовании определенного интеграла

Продолжим рассмотрение вопроса предыдущего параграфа. В каждом из отрезков возьмем по точке, которые обозначим

в каждой из этих точек вычислим значение функции Составим сумму

Эта сумма называется интегральной суммой для функции на отрезке Так как при произвольном принадлежащем отрезку будет и все то

следовательно,

или

Геометрический смысл последнего неравенства при состоит в том, что фигура, площадь которой равна ограничена ломаной, заключенной между «вписанной» ломаной и «описанной» ломаной.

Сумма зависит от способа разделения отрезка на отрезки и от выбора точек внутри получающихся отрезков.

Обозначим теперь через наибольшую из длин отрезков Рассмотрим различные разбиения отрезка на отрезки такие, что — Очевидно, что при этом число отрезков в разбиении стремится к бесконечности. Для каждого разбиения, выбрав соответствующие значения можно составить интегральную сумму

Рассмотрим пекоторую последовательность разбиений, при которых при этом При каждом разбиении выбираем значения Предположим, что эта последовательность интегральных сумм*) стремится к некоторому пределу

Теперь мы можем сформулировать следующее

Определение 1. Если при любых разбиениях отрезка таких, что и при любом выборе точек на отрезках интегральная сумма

стремится к одному и тому же пределу то этот предел называют определенным интегралом от функции на отрезке и обозначают

Таким образом, по определению

Число называется нижним пределом интеграла, —верхним пределом интеграла. Отрезок называется отрезком интегрирования, —переменной интегрирования.

Определение 2. Если для функции предел (6) существует, то функцию называют интегрируемой на отрезке

Заметим, что нижняя интегральная сумма и верхняя интегральная сумма являются частными случаями интегральной суммы (5), поэтому если интегрируема, то нижняя и верхняя интегральные суммы стремятся к тому же пределу и потому на основании равенства (6) можем написать

Если построить график подынтегральной функции то в случае интеграл

будет численно равен площади так называемой криволинейной трапеции, ограниченной указанной кривой, прямыми и осью (рис. 214).

Поэтому если требуется вычислить площадь криволинейной трапеции, ограниченной кривой прямыми и осью то эта площадь вычисляется с помощью интеграла:

Докажем следующую важную теорему.

Теорема 1. Если функция непрерывна на отрезке то она интегрируется на этом отрезке.

Доказательство. Снова разобьем отрезок на отрезки Составим нижнюю и верхнюю интегральные суммы:

Для дальнейшего установим некоторые свойства верхних и нижних интегральных сумм.

Свойство 1. При увеличении числа отрезков, на которые мы разбиваем отрезок путем добавления новых точек деления, нижняя интегральная сумма может только возрастать, а верхняя интегральная сумма только убывать.

Доказательство.

Пусть отрезок разбит на отрезков путем добавления новых точек Если какой-то отрезок будет разбит на несколько отрезков, например, на отрезков, то в новой нижней интегральной сумме отрезку будет соответствовать слагаемых, которые~мы обозначим через . В сумме этому отрезку соответствует одно слагаемое Но для суммы и величины справедливо неравенство, аналогичное неравенству (4) § 1. Мы можем написать

Написав соответствующие неравенства для каждого отрезка и суммируя левые и правые части, получим

Свойство 1 доказано.

Свойство 2. Нижняя интегрируемая сумма (9) и верхняя интегральная сумма (10) при неограниченном увеличении числа отрезков путем добавления новых точек деления стремятся к некоторым пределам

Доказательство.

На основании неравенства (6) § 1 можем написать:

т. е. ограничена при всех На основании свойств монотонно возрастает при возрастании Следовательно, на основании теоремы 7 о пределах (см. § 5 гл. II) эта переменная величина имеет предел; обозначим его через

Аналогично устанавливается, что ограничена снизу и монотонно убывает. Следовательно, имеет предел, который мы обозначим через

Свойство 3. Если функция непрерывна на замкнутом отрезке то пределы и определенные в свойстве 2 при условии, что равны.

Этот общий предел обозначим через

Свойство 3. Если функция непрерывна на замкнутом отрезке то пределы и определенные в свойстве 2 при условии, что равны.

Этот общий предел обозначим через

Доказательство. Рассмотрим разность верхней и нижней интегральной суммы:

Обозначим через наибольшую из разностей -— при данном разбиении:

Можно доказать (на чем мы останавливаться не будем), что если функция непрерывна на замкнутом отрезке, то при любом

способе разбиения отрезка если только

Свойство непрерывной функции на замкнутом отрезке, выражаемое равенством (15), называется равномерной непрерывностью функции.

Итак, мы будем пользоваться теоремой: Непрерывная функция на замкнутом отрезке равномерно непрерывна на этом отрезке.

Вернемся к равенству (14). Каждую разность в правой части заменим не меньшей величиной Получаем неравенство

Переходя к пределу при получаем

т. e.

или что и требовалось доказать.

Свойство 4. Пусть —нижняя и верхняя интегральные суммы, соответствующие разбиениям отрезка на и соответственно на отрезков. Тогда имеет место неравенство

при любых

Доказательство. Рассмотрим разбиение отрезка на отрезков, где точками деления будут точки деления первого и второго разбиений.

На основании неравенства (3) § 1 имеем

На основании свойства имеем

Пользуясь соотношениями (20) и (21), можно расширить неравенство (19):

что и требовалось доказать.

Свойство 5. Если функция непрерывна на отрезке то при любой последовательности разбиений отрезка на отрезки не обязательно путем присоединения новых

точек деления, если только нижняя интегральная сумма и верхняя интегральная сумма стремятся к пределу определенному в свойстве 3.

Доказательство. Рассмотрим последовательность разбиений последовательности верхних интегральных сумм определенных в свойстве 2. При любых значениях (на основании неравенства (18)) можем написать

Переходя к пределу при на основании (15) можем написать

Аналогичным способом докажем Итак,

или

Рассмотрим предел разности Так как функция непрерывна на замкнутом отрезке то (так же как и при доказательстве свойства 3) докажем (см. равенство (16)), что

Перепишем последнее соотношение так:

На основании (22) каждая из разностей, стоящих в квадратных скобках, неотрицательна. Следовательно,

и окончательно получаем

что и требовалось доказать.

Теперь можно доказать и сформулированную выше теорему. Пусть непрерывна на отрезке Рассмотрим произвольную последовательность интегральных сумм такую, что — произвольная точка отрезка

Для данной последовательности разбиений рассмотрим соответствующие последовательности верхних и нижних интегральных сумм и Для каждого разбиения будут справедливы соотношения (2):

Переходя к пределу при и пользуясь равенствами (23) и теоремой 4 § 5 гл. II, получаем где предел, определенный в свойстве 3.

Этот предел, как уже говорилось выше, и называется определенным интегралом Итак, если непрерывна на отрезке то

Отметим, что среди разрывных функций есть как интегрируемые, Так и: неинтегрируемые.

Пример 10.

Вычислим интеграл

Решение:

Геометрически задача эквивалентна вычислению площади трапеции, ограниченной линиями (рис. 215).

Функция стоящая под знаком интеграла, непрерывна. Следовательно, для вычисления определенного интеграла мы вправе, как это было замечено выше, произвести разбиение отрезка произвольным способом и произвольно выбрать промежуточные точки Результат вычисления определенного интеграла не зависит от способа построения интегральной суммы — лишь бы шаг разбиения стремился к нулю.

Делим отрезок на равных отрезков.

Длина каждого частичного отрезка равна это число и будет

шагом разбиения. Точки деления имеют координаты В качестве точек возьмем левые концы каждого отрезка: Составим интегральную сумму (I). Так как

где Учитывая что (как сумма геометрической прогрессии), получим

Так как Итак,

Площадь (рис. 215) легко вычислить методами элементарной геометрии.

Результат получится тот же.

Пример 11.

Вычислить

Решение:

Данный интеграл равен площади криволинейной трапеции, ограниченной параболой ординатой и прямой (рис. 216).

Разобьем отрезок на равных частей точками

За точки возьмем крайние правые точки каждого из отрезков. Составим интегральную сумму:

Как известно, поэтому

Пример 12. Вычислить

Решение:

Пример 13.

Вычислить

Решение:

Снова разделим отрезок на равных частей: За точки возьмем левые крайние точки. Составим интегральную сумму:

Выражение в скобках есть геометрическая прогрессия со знаменателем и первым членом 1 поэтому Далее имеем ( По правилу Лопиталя Таким образом: т.е.

Замечание.

Только что рассмотренные примеры показывают, что непосредственное вычисление определенных интегралов как пределов интегральных сумм связано с большими трудностями. Даже в тех случаях, когда подынтегральные функции являются эчень простыми этот способ требует громоздких подсчетов. Нахождение же определенных интегралов от более сложных функций приводит к еще большим трудностям. Поэтому естественно возникает задача: найти практически удобный метод вычисления определенных интегралов. Этот метод, открытый Нью-гоном и Лейбницем, использует глубокую связь, существующую между интегрированием и дифференцированием.

Несобственный интеграл для чайников. Несобственные интегралы

Вы еще здесь? =) Нет, я никого не пытался запугать, просто тема несобственных интегралов – очень хорошая иллюстрация тому, как важно не запускать высшую математику и другие точные науки. Для освоения урока на сайте всё есть – в подробной и доступной форме, было бы желание….

Итак, начнем-с. Образно говоря, несобственный интеграл – это «продвинутый» определенный интеграл, и на самом деле сложностей с ними не так уж и много, к тому же у несобственного интеграла есть очень хороший геометрический смысл.

Что значит вычислить несобственный интеграл?

Вычислить несобственный интеграл – это значит, найти ЧИСЛО (точно так же, как в определенном интеграле), или доказать, что он расходится (то есть, получить в итоге бесконечность вместо числа).

Несобственные интегралы бывают двух видов.

Несобственный интеграл с бесконечным пределом (ами) интегрирования

Иногда такой несобственный интеграл называют несобственным интегралом первого рода . В общем виде несобственный интеграл с бесконечным пределом чаще всего выглядит так: . В чем его отличие от определенного интеграла? В верхнем пределе. Он бесконечный: .

Реже встречаются интегралы с бесконечным нижним пределом или с двумя бесконечными пределами: , и их мы рассмотрим позже – когда войдёте во вкус:)

Ну а сейчас разберём самый популярный случай . В подавляющем большинстве примеров подынтегральная функция непрерывна на промежутке , и этот важный факт следует проверять в первую очередь! Ибо если есть разрывы, то есть дополнительные нюансы. Для определённости предположим, что и тогда типичная криволинейная трапеция будет выглядеть так:


Обратите внимание, что она бесконечна (не ограничена справа), и несобственный интеграл численно равен её площади . При этом возможны следующие варианты:

1) Первая мысль, которая приходит в голову: «раз фигура бесконечная, то », иными словами, площадь тоже бесконечна. Так быть может. В этом случае говорят, что несобственный интеграл расходится .

2) Но . Как это ни парадоксально прозвучит, площадь бесконечной фигуры может равняться… конечному числу! Например: . Может ли так быть? Запросто. Во втором случае несобственный интеграл сходится .

3) О третьем варианте чуть позже.

В каких случаях несобственный интеграл расходится, а в каком сходится? Это зависит от подынтегральной функции , и конкретные примеры мы очень скоро рассмотрим.

А что будет, если бесконечная криволинейная трапеция расположена ниже оси? В этом случае, несобственный интеграл (расходится) либо равен конечному отрицательному числу.

Таким образом, несобственный интеграл может быть отрицательным .

Важно! Когда Вам для решения предложен ЛЮБОЙ несобственный интеграл, то, вообще говоря, ни о какой площади речи не идет и чертежа строить не нужно . Геометрический смысл несобственного интеграла я рассказал только для того, чтобы легче было понять материал.

Коль скоро, несобственный интеграл очень похож на определенный интеграл, то вспомним формулу Ньютона- Лейбница: . На самом деле формула применима и к несобственным интегралам, только ее нужно немного модифицировать. В чем отличие? В бесконечном верхнем пределе интегрирования: . Наверное, многие догадались, что это уже попахивает применением теории пределов, и формула запишется так: .

В чем отличие от определенного интеграла? Да ни в чем особенном! Как и в определенном интеграле, нужно уметь находить первообразную функцию (неопределенный интеграл), уметь применять формулу Ньютона-Лейбница. Единственное, что добавилось – это вычисление предела. У кого с ними плохо, изучите урок Пределы функций. Примеры решений , ибо лучше поздно, чем в армии.

Рассмотрим два классических примера:

Пример 1

Для наглядности я построю чертеж, хотя, еще раз подчеркиваю, на практике строить чертежи в данном задании не нужно .

Подынтегральная функция непрерывна на полуинтервале , значит, всё нормально и несобственный интеграл можно вычислить «штатным» методом.

Применение нашей формулы и решение задачи выглядит так:

То есть, несобственный интеграл расходится, и площадь заштрихованной криволинейной трапеции равна бесконечности.

В рассмотренном примере у нас простейший табличный интеграл и такая же техника применения формулы Ньютона-Лейбница, как в определенном интеграле. Но применятся эта формула под знаком предела. Вместо привычной буквы «динамической» переменной выступает буква «бэ». Это не должно смущать или ставить в тупик, потому что любая буква ничем не хуже стандартного «икса».

Если Вам не понятно почему при , то это очень плохо, либо Вы не понимаете простейшие пределы (и вообще не понимаете, что такое предел), либо не знаете, как выглядит график логарифмической функции. Во втором случае посетите урок Графики и свойства элементарных функций .

При решении несобственных интегралов очень важно знать, как выглядят графики основных элементарных функций!

Чистовое оформление задания должно выглядеть примерно так:

! При оформлении примера всегда прерываем решение, и указываем, что происходит с подынтегральной функцией непрерывна она на промежутке интегрирования или нет . Этим мы идентифицируем тип несобственного интеграла и обосновываем дальнейшие действия.

Пример 2

Вычислить несобственный интеграл или установить его расходимость.

Выполним чертеж:

Во-первых, замечаем следующее: подынтегральная функция непрерывна на полуинтервале . Гуд. Решаем с помощью формулы :

(1) Берем простейший интеграл от степенной функции (этот частный случай есть во многих таблицах). Минус лучше сразу вынести за знак предела, чтобы он не путался под ногами в дальнейших вычислениях.

(2) Подставляем верхний и нижний пределы по формуле Ньютона-Лейбница.

(3) Указываем, что при (Господа, это уже давно нужно понимать) и упрощаем ответ.

Вот здесь площадь бесконечной криволинейной трапеции равна конечному числу! Невероятно, но факт.

Чистовое оформление примера должно выглядеть примерно так:

Подынтегральная функция непрерывна на

Что делать, если вам встретится интеграл наподобие – с точкой разрыва на интервале интегрирования? Это говорит о том, что в примере опечатка (вероятнее всего) , либо о продвинутом уровне обучения. В последнем случае, в силу свойства аддитивности , следует рассмотреть два несобственных интеграла на промежутках и и затем разобраться с суммой.

Иногда вследствие опечатки либо умысла несобственного интеграла может вовсе не существовать , так, например, если в знаменатель вышеуказанного интеграла поставить квадратный корень из «икс», то часть промежутка интегрирования вообще не войдёт в область определения подынтегральной функции.

Более того, несобственного интеграла может не существовать даже при всём «видимом благополучии». Классический пример: . Несмотря на определённость и непрерывность косинуса, такого несобственного интеграла не существует! Почему? Всё очень просто, потому что:
– не существует соответствующего предела .

И такие примеры пусть редко, но встречаются на практике! Таким образом, помимо сходимости и расходимости, есть ещё и третий исход решения с полноправным ответом: «несобственного интеграла не существует».

Следует также отметить, что строгое определение несобственного интеграла даётся именно через предел, и желающие могут ознакомиться с ним в учебной литературе. Ну а мы продолжаем практическое занятие и переходим к более содержательным задачам:

Пример 3

Вычислить несобственный интеграл или установить его расходимость.

Сначала попытаемся найти первообразную функцию (неопределенный интеграл). Если нам не удастся этого сделать, то несобственный интеграл мы, естественно, тоже не решим.

На какой из табличных интегралов похожа подынтегральная функция? Напоминает она арктангенс: . Из этих соображений напрашивается мысль, что неплохо бы в знаменателе получить квадрат. Делается это путем замены.

Проведем замену:

Неопределенный интеграл найден, константу в данном случае добавлять не имеет смысла.

На черновике всегда полезно выполнить проверку, то есть продифференцировать полученный результат:

Получена исходная подынтегральная функция, значит, неопределенный интеграл найден правильно.

Теперь находим несобственный интеграл:

(1) Записываем решение в соответствии с формулой . Константу лучше сразу вынести за знак предела, чтобы она не мешалась в дальнейших вычислениях.

(2) Подставляем верхний и нижний пределы в соответствии с формулой Ньютона-Лейбница. Почему при ? Смотрите график арктангенса в уже неоднократно рекомендованной статье.

(3) Получаем окончательный ответ. Тот факт, что полезно знать наизусть.

Продвинутые студенты могут не находить отдельно неопределенный интеграл, и не использовать метод замены, а использовать метод подведения функции под знак дифференциала и решать несобственный интеграл «сразу». В этом случае решение должно выглядеть примерно так:

Подынтегральная функция непрерывна на .

Пример 4

Вычислить несобственный интеграл или установить его расходимость.

! Это типовой пример, и похожие интегралы встречаются очень часто. Хорошо его проработайте! Первообразная функция здесь находится методом выделения полного квадрата, более подробно с методом можно ознакомиться на уроке Интегрирование некоторых дробей .

Пример 5

Вычислить несобственный интеграл или установить его расходимость.

Этот интеграл можно решить подробно, то есть сначала найти неопределенный интеграл, проведя замену переменной. А можно решить «сразу» – подведением функции под знак дифференциала. У кого какая математическая подготовка.

Полные решения и ответы в конце урока.

Примеры решений несобственных интегралов с бесконечным нижним пределом интегрирования можно посмотреть на странице Эффективные методы решения несобственных интегралов . Там же разобран случай, когда оба предела интегрирования бесконечны.

Несобственные интегралы от неограниченных функций

Или несобственные интегралами второго рода . Несобственные интегралы второго рода коварно «шифруются» под обычный определенный интеграл и выглядят точно так же: Но, в отличие от определенного интеграла, подынтегральная функция терпит бесконечный разрыв (не существует): 1) в точке , 2) или в точке , 3) или в обеих точках сразу, 4) или даже на отрезке интегрирования. Мы рассмотрим первые два случая, для случаев 3-4 в конце статьи есть ссылка на дополнительный урок.

Сразу пример, чтобы было понятно: . Вроде бы это определенный интеграл. Но на самом деле – это несобственный интеграл второго рода, если мы подставим в подынтегральную функцию значение нижнего предела , то знаменатель у нас обращается в ноль, то есть подынтегральной функции просто не существует в этой точке!

Вообще при анализе несобственного интеграла всегда нужно подставлять в подынтегральную функцию оба предела интегрирования . В этой связи проверим и верхний предел: . Здесь всё хорошо.

Криволинейная трапеция для рассматриваемой разновидности несобственного интеграла принципиально выглядит так:

Здесь почти всё так же, как в интеграле первого рода.

Наш интеграл численно равен площади заштрихованной криволинейной трапеции, которая не ограничена сверху. При этом могут быть два варианта*: несобственный интеграл расходится (площадь бесконечна) либо несобственный интеграл равен конечному числу (то есть, площадь бесконечной фигуры – конечна!).

* по умолчанию привычно полагаем, что несобственный интеграл существует

Осталось только модифицировать формулу Ньютона-Лейбница. Она тоже модифицируется с помощью предела, но предел стремится уже не к бесконечности, а к значению справа. Легко проследить по чертежу: по оси мы должны бесконечно близко приблизиться к точке разрыва справа .

Посмотрим, как это реализуется на практике.

Пример 6

Вычислить несобственный интеграл или установить его расходимость.

Подынтегральная функция терпит бесконечный разрыв в точке (не забываем устно или на черновике проверить, всё ли нормально с верхним пределом!)

Сначала вычислим неопределенный интеграл:

Замена:

У кого возникли трудности с заменой, обратитесь к уроку Метод замены в неопределенном интеграле .

Вычислим несобственный интеграл:

(1) Что здесь нового? По технике решения практически ничего. Единственное, что поменялось, это запись под значком предела: . Добавка обозначает, что мы стремимся к значению справа (что логично – см. график). Такой предел в теории пределов называют односторонним пределом . В данном случае у нас правосторонний предел .

(2) Подставляем верхний и нижний предел по формуле Ньютона Лейбница.

(3) Разбираемся с при . Как определить, куда стремится выражение? Грубо говоря, в него нужно просто подставить значение , подставляем три четверти и указываем, что . Причесываем ответ.

В данном случае несобственный интеграл равен отрицательному числу. В этом никакого криминала нет, просто соответствующая криволинейная трапеция расположена под осью .

А сейчас два примера для самостоятельного решения.

Пример 7

Вычислить несобственный интеграл или установить его расходимость.

Пример 8

Вычислить несобственный интеграл или установить его расходимость.

Если подынтегральной функции не существует в точке

Бесконечная криволинейная трапеция для такого несобственного интеграла принципиально выглядит следующим образом.

Определенный интеграл как предел интегральной суммы

может существовать (т.е. иметь определенное конечное значение) лишь при выполнении условий


Если хотя бы одно из этих условий нарушено, то определение теряет смысл. Действительно, в случае бесконечного отрезка, например [a ; ) его нельзя разбить на п частей конечной длины
, которая к тому же с увеличением количества отрезков стремилась бы к нулю. В случае же неограниченной в некоторой точкес [a ; b ] нарушается требование произвольного выбора точки на частичных отрезках – нельзя выбрать=с , поскольку значение функции в этой точке не определено. Однако и для этих случаев можно обобщить понятие определенного интеграла, введя еще один предельный переход. Интегралы по бесконечным промежуткам и от разрывных (неограниченных) функций называют несобственными .

Определение.

Пусть функция
определена на промежутке [a ; ) и интегрируема на любом конечном отрезке [a ; b ], т.е. существует
для любого b > a . Предел вида
называютнесобственным интегралом первого рода (или несобственным интегралом по бесконечному промежутку) и обозначают
.

Таким образом, по определению,
=
.

Если предел справа существует и конечен, то несобственный интеграл
называютсходящимся . Если этот предел бесконечен, или не существует вообще, то говорят, что несобственный интеграл расходится .

Аналогично можно ввести понятие несобственного интеграла от функции
по промежутку (–; b ]:

=
.

А несобственный интеграл от функции
по промежутку (–; +) определяется как сумма введенных выше интегралов:

=
+
,

где а – произвольная точка. Этот интеграл сходится, если сходятся оба слагаемых, и расходится, если расходится хотя бы одно из слагаемых.

С геометрической точки зрения, интеграл
,
, определяет численное значение площади бесконечной криволинейной трапеции, ограниченной сверху графиком функции
, слева – прямой
, снизу – осью ОХ. Сходимость интеграла означает существование конечной площади такой трапеции и равенство ее пределу площади криволинейной трапеции с подвижной правой стенкой
.

На случай интеграла с бесконечным пределом можно обобщить и формулу Ньютона-Лейбница :

=
=F(+ ) – F(a ),

где F(+ ) =
. Если этот предел существует, то интеграл сходится, в противном случае – расходится.

Мы рассмотрели обобщение понятия определенного интеграла на случай бесконечного промежутка.

Рассмотрим теперь обобщение для случая неограниченной функции.

Определение

Пусть функция
определена на промежутке [a ; b ), неограниченна в некоторой окрестности точки b , и непрерывна на любом отрезке
, где>0 (и, следовательно, интегрируема на этом отрезке, т.е.
существует). Предел вида
называетсянесобственным интегралом второго рода (или несобственным интегралом от неограниченной функции) и обозначается
.

Таким образом, несобственный интеграл от неограниченной в точке b функции есть по определению

=
.

Если предел справа существует и конечен, то интеграл называется сходящимся . Если конечного предела не существует, то несобственный интеграл называется расходящимся.

Аналогично можно определить несобственный интеграл от функции
имеющей бесконечный разрыв в точкеа :

=
.

Если функция
имеет бесконечный разрыв во внутренней точкес
, то несобственный интеграл определяется следующим образом

=
+
=
+
.

Этот интеграл сходится, если сходятся оба слагаемых, и расходится, если расходится хотя бы одно слагаемое.

С геометрической точки зрения, несобственный интеграл от неограниченной функции также характеризует площадь неограниченной криволинейной трапеции:

Поскольку несобственный интеграл выводится путем предельного перехода из определенного интеграла, то все свойства определенного интеграла могут быть перенесены (с соответствующими уточнениями) на несобственные интеграла первого и второго рода.

Во многих задачах, приводящих к несобственным интегралам, не обязательно знать, чему равен этот интеграл, достаточно лишь убедиться в его сходимости или расходимости. Для этого используют признаки сходимости . Признаки сходимости несобственных интегралов:

1) Признак сравнения .

Пусть для всех х

. Тогда, если
сходится, то сходится и
, причем

. Если
расходится, то расходится и
.

2) Если сходится
, то сходится и
(последний интеграл в этом случае называетсяабсолютно сходящимся ).

Признаки сходимости и расходимости несобственных интегралов от неограниченных функций аналогичны сформулированным выше.

Примеры решения задач.

Пример 1.

а)
; б)
; в)

г)
; д)
.

Решение.

а) По определению имеем:

.

б) Аналогично

Следовательно, данный интеграл сходится и равен .

в) По определению
=
+
, причем,а – произвольное число. Положим в нашем случае
, тогда получим:

Данный интеграл сходится.

Значит, данный интеграл расходится.

д) Рассмотрим
. Чтобы найти первообразную подынтегральной функции, необходимо применить метод интегрирования по частям. Тогда получим:

Поскольку ни
, ни
не существуют, то не существует и

Следовательно, данный интеграл расходится.

Пример 2.

Исследовать сходимость интеграла в зависимости от п .

Решение.

При
имеем:

Если
, то
и. Следовательно, интеграл расходится.

Если
, то
, а
, тогда

=,

Следовательно, интеграл сходится.

Если
, то

следовательно, интеграл расходится.

Таким образом,

Пример 3.

Вычислить несобственный интеграл или установить его расходимость:

а)
; б)
; в)
.

Решение.

а) Интеграл
является несобственным интегралом второго рода, поскольку подынтегральная функция
не ограничена в точке

. Тогда, по определению,

.

Интеграл сходится и равен .

б) Рассмотрим
. Здесь также подынтегральная функция не ограничена в точке
. Поэтому, данный интеграл – несобственный второго рода и по определению,

Следовательно, интеграл расходится.

в) Рассмотрим
. Подынтегральная функция
терпит бесконечный разрыв в двух точках:
и
, первая из которых принадлежит промежутку интегрирования
. Следовательно, данный интеграл – несобственный второго рода. Тогда, по определению

=

=

.

Следовательно, интеграл сходится и равен
.

Тема НЕСОБСТВЕННЫЕ ИНТЕГРАЛЫ

В теме «Определенный интеграл» было рассмотрено понятие определенного интеграла для случая конечного промежутка
и ограниченной функции
(см. теорему 1 из §3). Теперь займемся обобщением этого понятия для случаев бесконечного промежутка и неограниченной функции. Необходимость такого обобщения показывают, например, такие ситуации.

1. Если, используя формулу для длины дуги, попытаться вычислить длину четверти окружности
,
, то придем к интегралу от неограниченной функции:

, где
.

2. Пусть тело массой
движется по инерции в среде с силой сопротивления
, где
– скорость тела. Используя второй закон Ньютона (
, где
ускорение), получим уравнение:
, где
. Нетрудно показать, что решением этого (дифференциального!) уравнения является функция
Если нам потребуется вычислить путь, пройденный телом до полной остановки, т.е. до момента, когда
, то придем к интегралу по бесконечному промежутку:

I Определение

Пусть функция
определена и непрерывна на промежутке
. Тогда для любого
она интегрируема на промежутке
, то есть существует интеграл
.

Определение 1 . Конечный или бесконечный предел этого интеграла при
называют несобственным интегралом 1-го рода от функции
по промежутку
и обозначают символом
. При этом, если указанный предел конечен, то несобственный интеграл называют сходящимся, в противном случае (
или не существует) – расходящимся.

Итак, по определению

Примеры

2.
.

3.
– не существует.

Несобственный интеграл из примера 1 сходится, в примерах 2 и 3 интегралы расходятся.

II Формула Ньютона – Лейбница для несобственного интеграла первого рода

Пусть
– некоторая первообразная для функции
(сущест-вует на
, т.к.
– непрерывна). Тогда

Отсюда ясно, что сходимость несобственного интеграла (1) равносильна существованию конечного предела
. Если этот предел обозначить
, то можно написать для интеграла (1) формулу Ньютона-Лейбница:

, где
.

Примеры .

5.
.

6. Более сложный пример:
. Сначала найдем первообразную:

Теперь можем найти интеграл , учитывая, что

:

III Свойства

Приведем ряд свойств несобственного интеграла (1), которые вытекают из общих свойств пределов и определенного интеграла:


IV Другие определения

Определение 2 . Если
непрерывна на
, то

.

Определение 3 . Если
непрерывна на
, то принимают по определению

(– произвольное),

причем несобственный интеграл в левой части сходится, если только оба ин-теграла в правой части сходятся.

Для этих интегралов, как и для интеграла (1) можно написать соответствующие формулы Ньютона – Лейбница.

Пример 7 .

§2. Признаки сходимости несобственного интеграла 1-го рода

Чаще всего несобственный интеграл вычислить по определению не-возможно, поэтому используют приближенное равенство

(для больших ).

Однако, это соотношение имеет смысл лишь для сходящихся интегралов. Необходимо иметь методы выяснения поведения интеграла минуя определение.

I Интегралы от положительных функций

Пусть
на
. Тогда определенный интеграл
как функция верхнего предела есть функция возрастаю-щая (это следует из общих свойств определенного интеграла).

Теорема 1 . Несобственный интеграл 1 го рода от неотрицательной функ-ции сходится тогда и только тогда, когда функция
остается ограниченной при увеличении.

Эта теорема – следствие общих свойств монотонных функций. Практического смысла теорема почти не имеет, но позволяет получить т.н. признаки сходимости.

Теорема 2 (1-й признак сравнения). Пусть функции
и
непре-рывны на
и удовлетворяют неравенству
. Тогда:

1) если интеграл
сходится, то и
сходится;

2) если интеграл
расходится, то и
расходится.

Доказательство . Обозначим:
и
. Так как
, то

. Пусть интеграл
сходится, тогда (в силу теоремы 1) функция
‒ ограничена. Но тогда и
ограничена, а значит, интеграл
тоже сходится. Аналогично доказывается и вторая часть теоремы.

Этот признак не применим в случае расходимости интеграла от
или сходимости интеграла от
. Этот недостаток отсутствует у 2-го признака сравнения.

Теорема 3 (2-й признак сравнения). Пусть функции
и
непрерывны и неотрицательны на
. Тогда, если
при
, то несобственные интегралы
и
сходятся или расходятся одновременно.

Доказательство . Из условия теоремы получим такую цепочку равно-сильных утверждений:

, ,


.

Пусть, например,
. Тогда:

Применим теорему 2 и свойство 1) из §1 и получим утверждение теоремы 3.

В качестве эталонной функции, с которой сравнивают данную, высту-пает степенная функция
,
. Предлагаем студентам самим доказать, что интеграл

сходится при
и расходится при
.

Примеры . 1.
.

Рассмотрим подынтегральную функцию на промежутке
:

,
.

Интеграл
сходится, ибо
. По 2-му признаку сравнения сходится и интеграл
, а в силу свойства 2) из §1 сходится и исход-ный интеграл.

2.
.

Так как
, тоcуществует
такое, что при

. Для таких значений переменной:

Известно, что логарифмическая функция растет медленнее степенной, т.е.

,

а значит, начиная с некоторого значения переменной, эта дробь меньше 1. Поэтому

.

Интеграл сходится как эталонный. В силу 1-го признака сравнения сходится и
. Применяя 2-й признак, получим, что и интеграл
сходится. И снова свойство 2) из §1 доказывает сходимость исходного интеграла.

Как находить определенный интеграл для чайников. Решение определенного интеграла онлайн

Примеры вычисления неопределённых интегралов

Вычисление интеграла по таблице

Интегрирование подстановкой:

Примеры вычисления интегралов

Основная формула Ньютона – Лейбница

Вычисления подстановкой

Глава 4 Дифференциальные уравнения.

Дифференциальным уравнением называется уравнение, связывающее между собой независимую переменную х , искомую функции у и ее производные или дифференциалы.

Символически дифференцированное уравнение записывается так:

Дифференциальное уравнение называется обыкновенным , если искомая функция зависит от одного независимого переменного.

Порядком дифференциального уравнения называется порядок старшей производной (или дифференциала), входящей в данное уравнение.

Решением (или интегралом ) дифференциального уравнения называется такая функция, которая обращает это уравнение в тождество.

Общим решением (или общим интегралом ) дифференциального уравнения называется такое решение, в которое входит столько независимых произвольных постоянных, каков порядок уравнения. Так, общее решение дифференциального уравнения первого порядка содержит одну произвольную постоянную.

Частным решением дифференциального уравнения называется решение, полученное из общего при различных числовых значениях произвольных постоянных. Значения произвольных постоянных находятся при определенных начальных значениях аргумента и функции.

График частного решения дифференциального уравнения называется интегральной кривой.

Общему решению дифференциального уравнения соответствует совокупность (семейство) всех интегральных кривых.

Дифференциальным уравнением первого порядка называется уравнение, в которое входят производные (или дифференциалы) не выше первого порядка.

Дифференциальным уравнением с разделяющимися переменными называется уравнение вида

Для решения этого уравнения нужно сначала разделить переменные:

а затем проинтегрировать обе части полученного равенства:

1. Найти общее решение уравнения

o Разделив переменные имеем

Интегрируя обе части полученного уравнения:

Так как произвольная постоянная С может принимать любые числовые значения, то для удобства дальнейших преобразований вместо C мы написали (1/2) lnC. Потенцируя последнее равенство получим

Это и есть общее решение данного уравнения.

Литература

В. Г. Болтянский, Что такое дифференцирование, «Популярные лекции по математике»,

Выпуск 17, Гостехиздат 1955 г., 64 стр.

В. А. Гусев, А. Г. Мордкович «Математика»

Г. М. Фихтенгольц «Курс дифференциального и интегрального исчисления», том 1

В. М. Бородихин, Высшая математика, учеб. пособие, ISBN 5-7782-0422-1.

Никольский С. М. Глава 9. Определенный интеграл Римана // Курс математического анализа. – 1990. – Т. 1.

Ильин В. А., Позняк, Э. Г. Глава 6. Неопределенный интеграл // Основы математического анализа. – 1998. – Т. 1. – (Курс высшей математики и математической физики).

Демидович Б.П. Отдел 3. Неопределенный интеграл // Сборник задач и упражнений по математическому анализу. – 1990. – (Курс высшей математики и математической физики).

Валуцэ И.И., Дилигул Г.Д. Математика для техникумов на базе средней школы: Учеб.пособие-2-е изд.перераб. и доп. М.6Наука. 1989

Колягин Ю.М. Яковлев Г.Н. математика для техникумов. Алгебра и начала анализа 1 и 2 часть. Издательство «Наукка» М., 1981г.

Щипачев В.С. Задачи по высшей математике: Учеб. Пособие для вузов. Высш. Шк. 1997г.

Богомолов Н.В практические занятия по математике: учеб. Пособие для техникумов. Высш. Шк 1997г.

Данный калькулятор позволяет решить определенный интеграл онлайн. По сути, вычисление определенного интеграла – это нахождение числа, которое равно площади под графиком функции. Для решения необходимо задать границы интегрирования и интегрируемую функцию. После интегрирования система найдет первообразную для заданной функции, вычислит её значения в точках границах интегрирования, найдет их разность, что и будет являться решением определенного интеграла. Чтобы решить неопределенный интеграл вам необходимо воспользоваться похожим онлайн калькулятором, который находится на нашем сайте по ссылке – Решить неопределенный интеграл .

Мы позволяем вычислить определенный интеграл онлайн быстро и надежно. Вы получите всегда верное решение. Причем для табличных интегралов ответ будет представляться в классическом виде, то есть выражаться через известные константы, такие как число “пи”, “экспонента” и т.д. Все вычисления полностью бесплатны и не требуют регистрации. Решая определенный интеграл у нас, вы избавите себя от трудоемких и сложных вычислений, либо решив интеграл самостоятельно – вы сможете проверить полученное вами решение.

Онлайн сервис на сайт позволяет находить решение определенного интеграла онлайн . Решение проводится автоматически на сервере и в течении нескольких секунд пользователю выдается результат. Все онлайн сервисы на сайте абсолютно бесплатны, а решение выдается в удобном и понятном виде. Также нашим преимуществом является, что мы предоставляем возможность пользователю ввести границы интегрирования, в том числе и пределы интегрирования: минус и плюс бесконечность. Таким образом, решить определенный интеграл становится просто, быстро и качественно. Важно, что сервер позволяет вычислять определенные интегралы онлайн сложных функций, решение которых на иных онлайн-сервисах часто является невозможным ввиду несовершенства их систем. Мы предоставляем очень простой и интуитивно понятный механизм для ввода функций и возможность выбора переменной интегрирования, для чего вам не приходится переводить заданную в одной переменной функцию в другую, исключая связанные с этим ошибки и опечатки. Также на странице даны ссылки на теоретические статьи и таблицы по решению определенных интегралов. Всё в совокупоности позволит вам вычислять определенный интеграл онлайн очень быстро и при желании найти и разобраться с теорией решения определенных интегралов. На http://сайт вы также можете переходить на другие сервисы: онлайн решение пределов, производных, суммы рядов. Перейти же на вкладку решения неопределенных интегралов онлайн совсем просто – ссылка находится в ряду среди полезных ссылок. Более того, сервис постоянно совершенствуется и развивается, и с каждым днем появляются всё новые и новые возможности и усовершенствования. Решайте определенные интегралы вместе с нами! Все онлайн сервисы доступны даже незарегистрировшимся пользователям и абсолютно бесплатны.

Решая определенный интеграл у нас вы можете проверить своё собственное решение или избавиться от излишних трудоемких вычислений и довериться высокотехнологичной автоматизированной машине. Вычисляемая на сервисе точность удовлетворит практически любые инженерные нормы. Часто для многих табличных определенных интегралов результат выдается в точном выражении (используя общеизвестные константы и неэлементарные функции).

Решение интегралов – задача легкая, но только для избранных. Эта статья для тех, кто хочет научиться понимать интегралы, но не знает о них ничего или почти ничего. Интеграл… Зачем он нужен? Как его вычислять? Что такое определенный и неопределенный интегралы?

Если единственное известное вам применение интеграла – доставать крючком в форме значка интеграла что-то полезное из труднодоступных мест, тогда добро пожаловать! Узнайте, как решать простейшие и другие интегралы и почему без этого никак нельзя обойтись в математике.

Изучаем понятие « интеграл»

Интегрирование было известно еще в Древнем Египте. Конечно, не в современном виде, но все же. С тех пор математики написали очень много книг по этой теме. Особенно отличились Ньютон и Лейбниц , но суть вещей не изменилась.

Как понять интегралы с нуля? Никак! Для понимания этой темы все равно понадобятся базовые знания основ математического анализа. Сведения о пределах и производных , необходимые и для понимания интегралов, уже есть у нас в блоге.

Неопределенный интеграл

Пусть у нас есть какая-то функция f(x) .

Неопределенным интегралом функции f(x) называется такая функция F(x) , производная которой равна функции f(x) .

Другими словами интеграл – это производная наоборот или первообразная. Кстати, о том, как вычислять производные, читайте в нашей статье.


Первообразная существует для всех непрерывных функций. Также к первообразной часто прибавляют знак константы, так как производные функций, различающихся на константу, совпадают. Процесс нахождения интеграла называется интегрированием.

Простой пример:

Чтобы постоянно не высчитывать первообразные элементарных функций, их удобно свести в таблицу и пользоваться уже готовыми значениями.

Полная таблица интегралов для студентов


Определенный интеграл

Имея дело с понятием интеграла, мы имеем дело с бесконечно малыми величинами. Интеграл поможет вычислить площадь фигуры, массу неоднородного тела, пройденный при неравномерном движении путь и многое другое. Следует помнить, что интеграл – это сумма бесконечно большого количества бесконечно малых слагаемых.

В качестве примера представим себе график какой-нибудь функции.


Как найти площадь фигуры, ограниченной графиком функции? С помощью интеграла! Разобьем криволинейную трапецию, ограниченную осями координат и графиком функции, на бесконечно малые отрезки. Таким образом фигура окажется разделена на тонкие столбики. Сумма площадей столбиков и будет составлять площадь трапеции. Но помните, что такое вычисление даст примерный результат. Однако чем меньше и уже будут отрезки, тем точнее будет вычисление. Если мы уменьшим их до такой степени, что длина будет стремиться к нулю, то сумма площадей отрезков будет стремиться к площади фигуры. Это и есть определенный интеграл, который записывается так:


Точки а и b называются пределами интегрирования.


« Интеграл»

Кстати! Для наших читателей сейчас действует скидка 10% на любой вид работы

Правила вычисления интегралов для чайников

Свойства неопределенного интеграла

Как решить неопределенный интеграл? Здесь мы рассмотрим свойства неопределенного интеграла, которые пригодятся при решении примеров.

  • Производная от интеграла равна подынтегральной функции:

  • Константу можно выносить из-под знака интеграла:

  • Интеграл от суммы равен сумме интегралов. Верно также для разности:

Свойства определенного интеграла

  • Знак интеграла изменяется, если поменять местами пределы интегрирования:

  • При любых точках a , b и с :

Мы уже выяснили, что определенный интеграл – это предел суммы. Но как получить конкретное значение при решении примера? Для этого существует формула Ньютона-Лейбница:

Примеры решения интегралов

Ниже рассмотрим неопределенный интеграл и примеры с решением. Предлагаем самостоятельно разобраться в тонкостях решения, а если что-то непонятно, задавайте вопросы в комментариях.


Для закрепления материала посмотрите видео о том, как решаются интегралы на практике. Не отчаиваетесь, если интеграл не дается сразу. Обратитесь в профессиональный сервис для студентов, и любой тройной или криволинейный интеграл по замкнутой поверхности станет вам по силам.

Если определения из учебника слишком сложны и непонятны, прочитайте нашу статью. Мы постараемся максимально просто, “на пальцах” объяснить основные моменты такого раздела математики, как определенные интегралы. Как вычисляется интеграл, читайте в данной инструкции.

С геометрической точки зрения интеграл функции – это площадь фигуры, образуемой графиком данной функции и осью в пределах интегрирования. Запишите интеграл, проанализируйте функцию под интегралом: если подынтегральное выражение возможно упростить (сократить, вынести множитель на знак интеграла, разбить на два простых интеграла), сделайте это. Откройте таблицу интегралов, чтобы определить, производная какой функции стоит под интегралом. Ответ найден? Выпишете множитель, вынесенный за интеграл (если это имело место), запишите найденную из таблицы функцию, подставьте границы интеграла.


Для вычисления значения интеграла рассчитайте его значение в верхней границе и вычтите его значение в нижней границе. Разница – и есть искомая величина.


Чтобы проверить себя или хотя бы уяснить ход решения задачи на интегралы, удобно пользоваться онлайн-сервисом нахождения интегралов , однако прежде чем приступать к решению, ознакомьтесь с правилами ввода функций . Огромнейшее его преимущество в том, что здесь пошагово расписывается все решение задачи с интегралом.

Конечно, здесь рассмотрены лишь самые простые варианты интегралов – определенные, на самом деле разновидностей интегралов великое множество, изучаются они в курсе высшей математики, математического анализа и дифференциальных уравнений в ВУЗах для студентов технических специальностей.

Исчисление I – неопределенные интегралы

Показать уведомление для мобильных устройств Показать все заметки Скрыть все заметки

Похоже, вы используете устройство с “узкой” шириной экрана (, т.е. , вероятно, вы используете мобильный телефон). Из-за особенностей математики на этом сайте лучше всего просматривать в ландшафтном режиме.Если ваше устройство не находится в альбомном режиме, многие уравнения будут отображаться сбоку от вашего устройства (вы сможете прокручивать их, чтобы увидеть их), а некоторые пункты меню будут обрезаны из-за узкой ширины экрана.

Раздел 5-1: Неопределенные интегралы

В последних двух главах нам была задана функция \ (f \ left (x \ right) \), и мы спросили, какова производная этой функции.2} – 9x + c, \, \, \ hspace {0.25in} c {\ mbox {является константой}} \]

даст \ (f \ left (x \ right) \) при дифференцировании.

В последнем примере было два момента. Первым делом нужно было заставить вас задуматься о том, как решать эти задачи. Сначала важно помнить, что мы просто спрашиваем, что мы дифференцировали, чтобы получить данную функцию.

Другой момент – признать, что на самом деле существует бесконечное количество функций, которые мы могли бы использовать, и все они будут отличаться константой.

Теперь, когда мы разобрались с примером, давайте избавимся от некоторых определений и терминологии.

Определения

Для данной функции \ (f \ left (x \ right) \), антипроизводная от \ (f \ left (x \ right) \) является любой функцией \ (F \ left (x \ right) \) такой, что

\ [F ‘\ left (x \ right) = f \ left (x \ right) \]

Если \ (F \ left (x \ right) \) – любая антипроизводная от \ (f \ left (x \ right) \), то самая общая антипроизводная от \ (f \ left (x \ right) \) называется неопределенным интегралом и обозначается как

. \ [\ int {{е \ left (x \ right) \, dx}} = F \ left (x \ right) + c, \ hspace {0.25in} \, \, \, \, c {\ mbox {любая константа}} \]

В этом определении \ (\ int {{}} \) называется интегральным символом , \ (f \ left (x \ right) \) называется подынтегральным выражением , \ (x \) называется переменная интегрирования и «\ (c \)» называется постоянной интегрирования .

Обратите внимание, что часто мы просто говорим интеграл вместо неопределенного интеграла (или определенного интеграла в этом отношении, когда мы доходим до них). Из контекста проблемы будет ясно, что речь идет о неопределенном интеграле (или определенном интеграле).

Процесс нахождения неопределенного интеграла называется интегрированием или интегрированием \ (f \ left (x \ right) \) . Если нам нужно уточнить переменную интегрирования, мы скажем, что мы интегрируем \ (f \ left (x \ right) \) относительно \ (x \) . 4} + 3x – 9 \, dx}} \] Показать решение

Поскольку на самом деле здесь требуется самая общая антипроизводная, нам просто нужно повторно использовать окончательный ответ из первого примера.2} – 9x + c \]

Теперь сделаем пару предупреждений. Одна из наиболее распространенных ошибок, которые студенты делают с интегралами (как неопределенными, так и определенными), – это опускать dx в конце интеграла. Это обязательно! Думайте о знаке интеграла и dx как о скобках. Вы уже знаете и, вероятно, вполне довольны мыслью о том, что каждый раз, когда вы открываете скобку, вы должны закрывать ее. При использовании интегралов воспринимайте знак интеграла как «открытую скобку», а dx – как «закрывающую скобку».5} + c + 3x – 9 \ end {align *} \]

Вы интегрируете только то, что находится между знаком интеграла и dx . Каждый из приведенных выше интегралов заканчивается в разных местах, поэтому мы получаем разные ответы, потому что каждый раз мы интегрируем разное количество членов. Во втором интеграле «-9» находится за пределами интеграла, поэтому остается отдельно и не интегрируется. Точно так же в третьем интеграле «\ (3x – 9 \)» находится вне интеграла и поэтому остается в покое.

Знание, какие члены нужно интегрировать, – не единственная причина для записи \ (dx \) вниз.В разделе «Правило замены» мы фактически будем работать с \ (dx \) в задаче, и если у нас нет привычки записывать его, о нем легко забыть, и тогда мы получим неправильный ответ на этот этап.

Мораль заключается в том, чтобы убедиться и вставить \ (dx \)! На данном этапе это может показаться глупым поступком, но это просто необходимо, хотя бы по той причине, что знать, где заканчивается интеграл.

Кстати, обозначение \ (dx \) должно показаться вам немного знакомым.Мы видели такие вещи пару разделов назад. Мы назвали \ (dx \) дифференциалом в этом разделе, и да, это именно то, что он есть. \ (Dx \), завершающий интеграл, – не что иное, как дифференциал. 2} – 9w + c \ end {align *} \]

Изменение переменной интегрирования в интеграле просто изменяет переменную в ответе.Однако важно отметить, что при изменении переменной интегрирования в интеграле мы также изменили дифференциал (\ (dx \), \ (dt \) или \ (dw \)), чтобы он соответствовал новой переменной. Это более важно, чем мы можем себе представить на данный момент.

Другой вариант использования дифференциала в конце интеграла – сообщить нам, по какой переменной мы интегрируем. На данном этапе это может показаться несущественным, поскольку большинство интегралов, с которыми мы собираемся здесь работать, будут включать только одну переменную.Однако, если вы находитесь на пути к получению степени, который приведет вас к исчислению с несколькими переменными, это будет очень важно на этом этапе, поскольку в задаче будет более одной переменной. Вам нужно выработать привычку записывать правильный дифференциал в конце интеграла, чтобы, когда это станет важным в этих классах, вы уже будете иметь привычку записывать его.

Чтобы понять, почему это важно, взгляните на следующие два интеграла.

\ [\ int {{2x \, dx}} \ hspace {1.2} + c \]

Второй интеграл тоже довольно прост, но нам нужно быть осторожными. dx сообщает нам, что мы интегрируем \ (x \) ’s. Это означает, что мы интегрируем только \ (x \), которые находятся в подынтегральном выражении, а все другие переменные в подынтегральном выражении считаются константами. Тогда второй интеграл равен

. \ [\ int {{2t \, dx}} = 2tx + c \]

Таким образом, может показаться глупым всегда использовать dx , но это очень важная нотация, которая может привести к получению неправильного ответа, если мы не введем его.

Теперь есть несколько важных свойств интегралов, на которые мы должны обратить внимание.

Свойства неопределенного интеграла
  1. \ (\ displaystyle \ int {{к \, f \ left (x \ right) \, dx}} = k \ int {{f \ left (x \ right) \, dx}} \) где \ ( k \) – любое число. Итак, мы можем выделить мультипликативные константы из неопределенных интегралов.

    См. Раздел «Доказательство различных интегральных формул» в главе «Дополнительно», чтобы увидеть доказательство этого свойства.

  2. \ (\ displaystyle \ int {{- f \ left (x \ right) \, dx}} = – \ int {{f \ left (x \ right) \, dx}} \). Это действительно первое свойство с \ (k = – 1 \), поэтому доказательства этого свойства не приводятся.
  3. \ (\ Displaystyle \ int {{е \ влево (х \ вправо) \ pm г \ влево (х \ вправо) \, dx}} = \ int {{е \ влево (х \ вправо) \, dx}} \ pm \ int {{g \ left (x \ right) \, dx}} \). Другими словами, интеграл от суммы или разности функций – это сумма или разность отдельных интегралов.Это правило можно распространить на любое количество функций.

    См. Раздел «Доказательство различных интегральных формул» в главе «Дополнительно», чтобы увидеть доказательство этого свойства.

Обратите внимание, что когда мы работали с первым примером выше, мы использовали первое и третье свойство в обсуждении. Мы интегрировали каждый термин индивидуально, вернули все константы, а затем снова собрали все вместе с соответствующим знаком.

В приведенных выше свойствах не указаны интегралы от произведений и частных.Причина этого проста. Как и в случае с производными финансовыми инструментами, каждое из следующих действий НЕ будет работать.

\ [\ int {{f \ left (x \ right) g \ left (x \ right) \, dx}} \ ne \ int {{f \ left (x \ right) dx}} \ int {{g \ left (x \ right) \, dx}} \ hspace {0.75in} \ int {{\ frac {{f \ left (x \ right)}} {{g \ left (x \ right)}} \, dx }} \ ne \ frac {{\ int {{f \ left (x \ right) \, dx}}}} {{\ int {{g \ left (x \ right) \, dx}}}} \]

Что касается деривативов, у нас было правило продукта и правило частного, чтобы иметь дело с этими случаями. Однако с интегралами таких правил нет.Когда мы сталкиваемся с произведением и частным в интеграле, у нас будет множество способов справиться с ним, в зависимости от того, что такое подынтегральное выражение. 4} + 3x – 9 \), что было \ (f \ left (x \ right) \)? Показать решение

К этому моменту в этом разделе это простой вопрос.2} – 9x + c \]

В этом разделе мы продолжали вычислять один и тот же неопределенный интеграл во всех наших примерах. Целью этого раздела было не делать неопределенные интегралы, а вместо этого познакомить нас с обозначениями и некоторыми основными идеями и свойствами неопределенных интегралов. Следующая пара разделов посвящена фактическому вычислению неопределенных интегралов.

Неопределенные интегралы – исчисление 2

Если вы считаете, что контент, доступный через Веб-сайт (как определено в наших Условиях обслуживания), нарушает или несколько ваших авторских прав, сообщите нам, отправив письменное уведомление («Уведомление о нарушении»), содержащее в информацию, описанную ниже, назначенному ниже агенту.Если репетиторы университета предпримут действия в ответ на ан Уведомление о нарушении, оно предпримет добросовестную попытку связаться со стороной, которая предоставила такой контент средствами самого последнего адреса электронной почты, если таковой имеется, предоставленного такой стороной Varsity Tutors.

Ваше Уведомление о нарушении прав может быть отправлено стороне, предоставившей доступ к контенту, или третьим лицам, таким как в виде ChillingEffects.org.

Обратите внимание, что вы будете нести ответственность за ущерб (включая расходы и гонорары адвокатам), если вы существенно искажать информацию о том, что продукт или действие нарушает ваши авторские права.Таким образом, если вы не уверены, что контент находится на Веб-сайте или по ссылке с него нарушает ваши авторские права, вам следует сначала обратиться к юристу.

Чтобы отправить уведомление, выполните следующие действия:

Вы должны включить следующее:

Физическая или электронная подпись правообладателя или лица, уполномоченного действовать от их имени; Идентификация авторских прав, которые, как утверждается, были нарушены; Описание характера и точного местонахождения контента, который, по вашему мнению, нарушает ваши авторские права, в \ достаточно подробностей, чтобы позволить репетиторам университетских школ найти и точно идентифицировать этот контент; например нам требуется а ссылка на конкретный вопрос (а не только на название вопроса), который содержит содержание и описание к какой конкретной части вопроса – изображению, ссылке, тексту и т. д. – относится ваша жалоба; Ваше имя, адрес, номер телефона и адрес электронной почты; и Ваше заявление: (а) вы добросовестно считаете, что использование контента, который, по вашему мнению, нарушает ваши авторские права не разрешены законом, владельцем авторских прав или его агентом; (б) что все информация, содержащаяся в вашем Уведомлении о нарушении прав, является точной, и (c) под страхом наказания за лжесвидетельство, что вы либо владелец авторских прав, либо лицо, уполномоченное действовать от их имени.

Отправьте жалобу нашему уполномоченному агенту по адресу:

Чарльз Кон Varsity Tutors LLC
101 S. Hanley Rd, Suite 300
St. Louis, MO 63105

Или заполните форму ниже:

index-of.es/

 Название Размер
 Android / -
 Галерея искусств/                  -
 Атаки / -
 Переполнение буфера / -
 C ++ / -
 CSS / -
 Компьютер / -
 Конференции / -
 Растрескивание / -
 Криптография / -
 Базы данных / -
 Глубокая сеть / -
 Отказ в обслуживании/            -
 Электронные книги / -
 Перечисление / -
 Эксплойт / -
 Техники неудачной атаки / -
 Судебная экспертиза / -
 Галерея / -
 HTML / -
 Взломать / -
 Взлом-веб-сервер / -
 Взлом беспроводных сетей / -
 Взлом / -
 Генератор хешей / -
 JS / -
 Джава/                         -
 Linux / -
 Отмыкание/                  -
 Журналы / -
 Вредоносное ПО / -
 Метасплоит / -
 Разное / -
 Разное / -
 Протоколы сетевой безопасности / -
 Сеть / -
 ОПЕРАЦИОННЫЕ СИСТЕМЫ/                           -
 Другое / -
 PHP / -
 Perl / -
 Программирование / -
 Python / -
 RSS / -
 Rdbms / -
 Обратный инжиниринг/          -
 Рубин/                         -
 Сканирование сетей / -
 Безопасность/                     -
 Захват сеанса / -
 Снифферы / -
 Социальная инженерия/           -
 Поддерживает / -
 Системный взлом / -
 Инструменты/                        -
 Учебники / -
 UTF8 / -
 Unix / -
 Вариос-2 / -
 Варианты / -
 Видео/                       -
 Вирусы / -
 Окна / -
 Беспроводная связь / -
 Xml / -
 z0ro-Репозиторий-2 / -
 z0ro-Репозиторий-3 / -
 

2.3 + K` и произнеси прописью:

“Интеграл 3 x 2 относительно x равно x 3 + K . “

Знак интеграла

Знак `int` представляет собой удлиненную букву” S “, стоя для «суммы». (В старонемецком и английском языках буква «s» часто писалась с использованием этой удлиненной формы.) Позже мы увидим, что интеграл – это – сумма площадей бесконечно тонких прямоугольников.

«сумма» – это символ «сумма».Его можно использовать для конечных или бесконечных сумм.

int – это символ суммы бесконечного числа бесконечно малых областей (или других переменных).

Обозначение «длинное s» было введено Лейбницем, когда он разрабатывал концепции интеграции.

Другое обозначение интегралов

Примечание: Иногда мы пишем заглавную букву, чтобы обозначить интеграцию. Например, мы пишем F ( x ), чтобы означать интеграл f ( x ).3} {3} -5x + K`

Теперь мы узнаем несколько важных общих правил интеграции.

A. Интеграл константы

`intk dx = kx + K`

(k и K – константы.)

Интеграл постоянной – это постоянные времена x плюс константа.

Пример 3

Найдите `int4 dx`

Ответ

Используя наше новое правило, мы можем просто написать:

`int4 dx = 4x + K`

Всегда проверяйте по , дифференцируя , свой ответ, и вы должны вернуться к тому, о чем спрашивал вас, чтобы интегрировать.2 + 9) + К`

Пример 14

Для данного `y ‘= sqrt (2x + 1` найти функцию` y = f (x) `, который проходит через точку` (0,2) `.

Ответ

ПРИМЕЧАНИЕ: y означает производную от y, то есть (dy) / dx.

`y ‘= sqrt (2x + 1`

Так

`y = intsqrt (2x + 1) dx`

Этот вопрос требует, чтобы мы интегрировали и в процессе находили константу интегрирования .(3 // 2)) / 3 + 5 / 3`

– это обязательная функция.

Вот график ответа, который мы нашли в примере 14. Вы можете видеть, что он проходит через (2, 0).

График и , проходящий через (2, 0).

Примечание: в этой работе вы увидите «+ K» и «+ C». Наиболее учебники используют + C.

Рекомендуется всегда использовать . использовать + K , если вы отвечаете на вопросы электриков. проблемы.

% PDF-1.4 % 1 0 объект > эндобдж 3 0 obj > эндобдж 2 0 obj > эндобдж 4 0 obj > / ProcSet [/ PDF / ImageC] / XObject> >> / Повернуть 0 / StructParents 0 / Тип / Страница >> эндобдж 5 0 obj > / ProcSet [/ PDF / ImageC] / XObject> >> / Повернуть 0 / StructParents 1 / Тип / Страница >> эндобдж 6 0 obj > / Шрифт> / ProcSet [/ PDF / Text] / Свойства> >> >> / Повернуть 0 / StructParents 2 / TrimBox [0,0 0,0 531,0 666,0] / Тип / Страница >> эндобдж 7 0 объект > / Шрифт> / ProcSet [/ PDF / Text] >> / Повернуть 0 / StructParents 3 / TrimBox [0.0 0,0 531,0 666,0] / Тип / Страница >> эндобдж 8 0 объект > / Шрифт> / ProcSet [/ PDF / Text] >> / Повернуть 0 / StructParents 4 / TrimBox [0,0 0,0 531,0 666,0] / Тип / Страница >> эндобдж 9 0 объект > / Шрифт> / ProcSet [/ PDF / Text] >> / Повернуть 0 / StructParents 5 / TrimBox [0,0 0,0 531,0 666,0] / Тип / Страница >> эндобдж 10 0 obj > / Шрифт> / ProcSet [/ PDF / Text] >> / Повернуть 0 / StructParents 13 / TrimBox [0,0 0,0 531,0 666,0] / Тип / Страница >> эндобдж 11 0 объект > >> / Повернуть 0 / TrimBox [0.0 0,0 531,0 666,0] / Тип / Страница >> эндобдж 12 0 объект > / Шрифт> / ProcSet [/ PDF / Text] >> / Повернуть 0 / StructParents 14 / TrimBox [0,0 0,0 531,0 666,0] / Тип / Страница >> эндобдж 13 0 объект > / Шрифт> / ProcSet [/ PDF / Text] >> / Повернуть 0 / StructParents 15 / TrimBox [0,0 0,0 531,0 666,0] / Тип / Страница >> эндобдж 14 0 объект > / Шрифт> / ProcSet [/ PDF / Text] >> / Повернуть 0 / StructParents 16 / TrimBox [0,0 0,0 531,0 666,0] / Тип / Страница >> эндобдж 15 0 объект > / Шрифт> / ProcSet [/ PDF / Text] >> / Повернуть 0 / StructParents 17 / TrimBox [0.0 0,0 531,0 666,0] / Тип / Страница >> эндобдж 16 0 объект > / Шрифт> / ProcSet [/ PDF / Text] >> / Повернуть 0 / StructParents 18 / TrimBox [0,0 0,0 531,0 666,0] / Тип / Страница >> эндобдж 17 0 объект > / Шрифт> / ProcSet [/ PDF / Text] >> / Повернуть 0 / StructParents 19 / TrimBox [0,0 0,0 531,0 666,0] / Тип / Страница >> эндобдж 18 0 объект > / Шрифт> / ProcSet [/ PDF / Text] >> / Повернуть 0 / StructParents 20 / TrimBox [0,0 0,0 531,0 666,0] / Тип / Страница >> эндобдж 19 0 объект > ручей / Часть> BDC q 531.N ߽ ~ w.Mha

Страница не найдена – Tour Marocco 4×4

Recensioni

  • Мы прекрасно провели время в нашем туре по Марокко. Водитель был очень осторожен, машина была в безопасности, у нас была возможность побывать во многих разных местах, и … прочитайте больше нас впечатлили отели, в которых мы останавливались.

    Olivia M
    27.09.2016
  • Мы заканчиваем тур продолжительностью 8 дней / 7 ночей из Марракеша на юг Марокко, и мы очень довольны этим опытом, который также включает в себя… читать дальше ночь в заброшенном лагере Erg Chebbi. Очень гостеприимные и традиционные различные риады, в которых мы останавливались. Специальное предложение для нашего водителя / сопровождающего Сказано, всегда точно, пунктуально и профессионально! Спасибо также Алессандре, которая всегда была на связи с нами, чтобы знать, все ли в порядке. Это агентство однозначно рекомендуется для тура по самому аутентичному Марокко…

    claudio g
    01.06.2018
  • Мы заканчиваем тур продолжительностью 8 дней / 7 ночей из Марракеша на юг Марокко, и мы очень довольны этим опытом, который также включает в себя… читать дальше ночь в заброшенном лагере Erg Chebbi. Очень гостеприимные и традиционные различные риады, в которых мы останавливались. Специальное предложение для нашего водителя / сопровождающего Сказано, всегда точно, пунктуально и профессионально! Спасибо также Алессандре, которая всегда была на связи с нами, чтобы знать, все ли в порядке. Это агентство однозначно рекомендуется для тура по самому аутентичному Марокко

    . claudio g
    23.06.2018
  • Агентство Mobarak’s Sahara Experience Tours, гид, сопровождавший нас в нашем путешествии, с самого начала оказалось профессиональным и надежным.
    Я чувствую себя настоятельно рекомендованным этим агентством по нескольким причинам: доступность наших запросов на посещение, точность выбранного жилья (Riad), которое превзошло наши ожидания, и внимание, с которым наша поездка была организована там, где это на самом деле не так. ничего не упустил.
    Во время нашего путешествия из Марракеша к дюнам пустыни Эрг-Шебби мы путешествовали по разным природным ландшафтам, встречали множество мест и деревень и наслаждались марокканской культурой. Мобарак (очень красиво) и агентство смогли удовлетворить все наши требования, обеспечивая необходимый комфорт: удобный джип, сумка-холодильник для лекарств и воды, Wi-Fi, электричество в лагере на дюнах и покупка местных продуктов в места подходящие.

    Мы благодарим вас за прекрасный опыт и надеемся встретиться снова во время будущей поездки в Марокко.

    Анна, Пол и Луиджи.

    анна б
    31.05.2018
  • Прекрасное время в Сахаре Мерзуги, с Мобараком, моим гидом на верблюдах. Мы провели ночь в палатках в пустыне, и они накормили нас хорошей едой и чаем. На следующий день Мобарак показал мне деревню кочевников, и я получил отличное представление! Настоятельно рекомендуется! Спасибо!

    София Б
    01.09.2016
  • Первая поездка в Марокко началась в Марракеше и завершилась в Фесе.Мы связались с Tour Morocco 4×4, чтобы составить маршрут поездки, и Мобарак сразу стал mostratogentilissima … читать дальше, и мы подготовили отличную 5-дневную программу, чтобы пересечь горы высокого атласа, подняться в пустыню эрг-шебби, а затем отправиться в путь. за горами среднего Атласа до Феса. Обычно наши поездки всегда связаны с движением, тогда как в нашем случае это была программа. с первого дня поездки мы чувствовали себя комфортно с нашим частным гидом, который сказал, что на джипе 4×4 нас сопровождал, чтобы посетить чудеса своей земли.sayè молодой мальчик и легкий, что в дополнение к illustrarci достопримечательности сделали нас еще очень весело! Кроме того, очень хорошо говорит по-итальянски! Остановились во время экскурсии в красивых отелях, пообедали и поужинали в основе типичных блюд, всегда превосходных. Мы также спали под звездами между дюнами пустыни. незабываемый! сказал, что всегда присутствовал и был готов удовлетворить любую нашу просьбу. Организованную поездку однозначно рекомендую тем, кто хочет открыть для себя Марокко и познакомиться с его гостеприимным населением.После этого путешествия мы всегда будем приносить с собой воспоминания об ароматах, цветах и ​​эмоциях, которые люди, известные нам, смогли прислать нам! Тем не менее, спасибо организатору Мобараку за то, что он очень хорошо сказал нашему гиду, и за то, что, к сожалению, мы не знали. Большой привет, скоро. …

    alinealexandre
    01.02.2017
  • Мы провели 8 дней в Марокко, в Марракеше, ущелье Дадес, Мерзуге, Фесе и Шефшауэне. В любое время трасса была организована до совершенства, от выбора отелей до… прочитайте больше планирование мест для посещения и теплоту нашего гида-водителя САИДа, который всегда был осведомлен о нас, создавал веселую и уверенную атмосферу и рассказывал нам о своей культуре, должен был больше разделить поездку с другом. Даже гиды, которые были у нас в некоторых местах, были очень внимательны. Большое спасибо Othm avis, Ромео посоветует место назначения и посетит Марокко. Спасибо за все, увидимся :)…

    Сара б
    01.06.2018

Интегрирование по частям – неопределенные интегралы – x-engineering.org

Интегрирование по частям – это метод вычисления неопределенных интегралов с использованием дифференциала произведения двух функций.

Если у нас есть две функции, u и v , дифференциал их произведения будет:

\ [d (u \ cdot v) = u \ cdot dv + v \ cdot du \ tag {1} \ ]

Если мы проинтегрируем обе части математического выражения, мы получим:

\ [\ int d (u \ cdot v) = \ int \ left (u \ cdot dv + v \ cdot du \ right) \ tag { 2} \]

Мы знаем, что интеграл от производной функции дает саму функцию.Итак:

\ [\ int d (u \ cdot v) = u \ cdot v \ tag {3} \]

Кроме того, интеграл суммы двух функций является суммой интеграла каждой функции:

\ [ \ int \ left (u \ cdot dv + v \ cdot du \ right) = \ int u \ cdot dv + \ int v \ cdot du \ tag {4} \]

Замена уравнений (3) и (4) в уравнении (2) получаем:

\ [u \ cdot v = \ int u \ cdot dv + \ int v \ cdot du \ tag {5} \]

Преобразование уравнения (5) дает математическое выражение для интегрирования по частям :

\ [\ bbox [# FFFF9D] {\ int u \ cdot dv = u \ cdot v – \ int v \ cdot du} \ tag {6} \]

Пример 1 .Решите следующий интеграл, используя интегрирование по частям:

\ [\ int x \ cdot cos (x) dx \]

Как мы видим, интеграл содержит произведение двух функций: x и cos (x) . Мы собираемся заменить их на u и v .

Шаг 1 . Замените функции на u и v

Сначала мы заменим:

\ [u = x \ tag {7} \]

Применяя дифференцирование к уравнению (7), получаем:

\ [du = dx \ tag {8} \]

Во-вторых, мы заменяем:

\ [dv = cos (x) dx \ tag {9} \]

Применяя интегрирование к уравнению (9), получаем:

\ [\ int dv = \ int cos (x) dx \ tag {10} \] \ [v = sin (x) \ tag {11} \]

Шаг 2 .

Оставить комментарий